american education aids - higher education...

40
AMERICAN EDUCATION AIDS GRE - ANALYTICAL WRITING ABILITY MODEL ESSAYS ON ARGUMENTS (1 to 90)

Upload: hanguyet

Post on 29-Mar-2018

216 views

Category:

Documents


0 download

TRANSCRIPT

AMERICAN EDUCATION AIDSGRE - ANALYTICAL WRITING ABILITY

MODEL ESSAYS ON ARGUMENTS(1 to 90)

GRE - ANALYTICAL WRITING ABILITY

ANALYSIS OF ARGUMENTS

MODEL ESSAYS 1 - 90

An essay analyzing an ‘Argument’ is easier to compose than one analyzing an ‘Issue’. This is because, inthe case of an ‘Argument’, you have some concrete statements in front of you to analyze, and do not have tostart from scratch.

An ‘argument’ passage will consist of some facts, some research findings or some opinions of expertsfollowed by some conclusions or recommendations by the author of the argument.

You must recognize each of these and must differentiate among them. You should write a critique of onlythe last, namely the ‘conclusions or recommendations’. You should not try to question the correctness oraccuracy of the first three.

In narrating each ‘Argument’, the question setters have deliberately incorporated two orthree logical flaws in it. Their intention is to test whether you are capable of spotting these flawsand commenting on them logically.

An analysis of the pool of 242 Arguments for GRE, released by Educational Testing Service, reveals thatthe types of logical flaws that are repeatedly encountered are JUST 14 in number. It is these 14 flaws whichare incorporated in different contexts in different Argument topics.

The types of these 14 logical flaws are:

1. Concluding that because Action A caused Result X a few years ago, a similar action A nowwill cause a similar result X.

While analyzing an argument containing this flaw, you should point out that the relevant circumstancesmight have changed significantly in the last few years, and that the same action may not produce the sameresult now. In the context of the given narration, you should enumerate some of the possible ways in which thecircumstances might have changed in the intervening years.

2. Comparing the achievement record of Company A in a particular year (say, 1995) with abetter achievement record of Company B in some other year (say, 2001), and concluding thatCompany B is therefore more efficiently run than Company A.

While analyzing such an argument, you could point out that the year 1995 was perhaps bad for allcompanies, and that Company B, if it had existed then, might have fared equally badly, or even worse, thanCompany A in that year. You should also point out that, if the achievement record of Company A for 2001 isavailable, it may perhaps be seen that Company A had done even better than Company B in 2001, and istherefore the more efficient of the two. You could also state that these two companies may be in two totallydifferent types of industries and are not comparable at all on the basis of any single common parameter. Youmay give instances of difference in the nature of these two companies.

3. Concluding that, because Institution A (a college, university, school, town council, state, amagazine, a shop or a supermarket) had taken Action X (such as introducing a new syllabus, anew tax, a new regulation, introducing a new feature, reducing the price, stocking a new itemetc.) successfully, Institution B should also take the same Action X to achieve the same result.

You should point out that conditions are perhaps different in the two institutions, and an action whichwas successful in one may not prove to be equally successful in the other. In the context of the givennarration, you should enumerate a few possible differences in characteristics between the two institutions.

American Education Aids - GRE Arguments - Model Essays 3

4. Comparing the efficiency of two organizations A and B (such as two shops, two hospitals,two schools etc.), on the basis of just a single parameter (such as turnover, or the number ofpatients who had died, the number of prizes it won in a certain athletic competition etc.) andconcluding that one is more efficient than the other.

You should point out that a single parameter cannot be used to compare the overall relative efficiencies oftwo institutions, and that other factors (such as profit per employee, profit per dollar invested, or the numberof terminally ill patients who had been admitted, academic success of the students etc.) must also be takeninto consideration.

5. Trying to apply the result of a statistical study in a limited area to a much wider area or,conversely, trying to apply the result of a nationwide statistical study to a limited local area. Avariation of this is the extrapolation of the results of a study over a short period (say, six months)as valid for a much longer period (such as a decade or a century).

In the former cases, you should point out that conditions in the local area may be significantly differentfrom what is prevalent in the wider geographical area or vice versa, and that, therefore, the results of aparticular statistical study may not be true in both. In the latter case, you should point out how conditionsrelevant to the conclusion may have changed over a period over a period of time. In the context of the givennarration, you should enumerate a few possible differences in conditions that may arise.

6. Extrapolating the result of a statistical study on a small sample to a much biggerpopulation, or to a totally unrelated group (for example, the result of a medical experiment onrats being applied without modification to human beings).

You should point out that the sample that was studied was perhaps not typical of the general population,and that what was true of this biased sample may not be true of the total population. In the context of thegiven narration, you can enumerate a few possible ways in which the sample may differ from the generalpopulation.

7. Concluding from a percentage difference (such as 5% or 95%), but without the base figure,that the increase/decrease was either high or low.

You should point out that an increase of 90% over a base figure of 100 will be much less than an increaseof only 10% over a base figure of 10,000. Therefore, without knowing the base figure, it is not possible to judgewhether the increase/decrease was high or low. You can cite such figures (whether 100, 1000, 10,000 or amillion) in the context of the given narration.

8. Concluding that, because Action A will increase the sales revenue of a company, it will alsoincrease its profit. Or, conversely, a decrease in sales turnover will result in a decrease in profit.

You should point out that an increase in sales revenue need not always be attended by an increase inprofit, or vice versa. The steps taken to increase the turnover, such as opening new branches, appointing newstaff, incurring additional advertisement expenditure, increased transportation, increased interest on capitalemployed etc., may sometimes result in a decrease in overall profit. Similarly, austerity measures which willsave considerable unnecessary expenditure may sometimes result in a reduction in turnover even whileincreasing the net profit.

9. Arriving at a certain conclusion on the basis of the opinion expressed by a high percentage(such as 90%) of ‘those who responded’ to a survey.

You should ask the questions: What was the total population? What percentage among them wereapproached with a questionnaire or were interviewed? And what percent among these actually responded?

If only 10% of a total population were approached, and only 10% among them responded, then the opinionin favor of the proposed conclusion is that of only 1% of the total population, and it cannot be considered to bereflective of that of the whole population.

10. Arriving at a conclusion on the basis of the opinion of, or experiments on, ‘many’, ‘several’,or ‘a number of’ persons or subjects, without actually quantifying these numbers.

You must point out that these vague adjectives may mean any number (say, from 20 to 20,000) and that,unless they are specifically quantified, it is not possible to come to any reliable conclusion.

11. Concluding that because Event B followed Event A, Event B must have been caused byEvent A. (This is known as a ‘causal relationship’ between A and B).

American Education Aids - GRE Arguments - Model Essays 4

You must point out that what caused Event B might have no relationship at all to Event A, but wassomething totally different. In the context of the given narration, you can enumerate some of the otherprobable causes for Event B.

12. Arriving at a conclusion on the assumption that the reported number (of a particularillness such as headache, an accident etc.) is the same as the actual number.

You must point out that not everyone (who had that illness or who had been involved in an accident)might have reported for treatment at a hospital. In the context of the given narration, you may give possiblereasons for their action.

13. Concluding that only one of Event A or Event B can happen under the given circumstances,without considering the possibility that both events can happen simultaneously.

In such cases, you should, in the light of the given context, give examples of how both events can happensimultaneously.

14. Stating a possible outcome as an inevitable outcome.In the context of the given narration, you should point out how the project outcome may not materialize

under certain other conditions.

Every given Argument will contain two or three of the 14 types of flaws explained above, cleverlyincorporated within the context of the given narration. These flaws may not be apparent to you during the firstreading. When you read the narration again carefully, you should be able to spot them and comment on themas suggested above.

Each ‘Argument’ topic has been carefully drafted by the question-setters, and every word or phrase in ithas a specific reason for being there, and can help you to compose a cogent sentence or two in your essay.

Each of the following model essays is a ‘first draft’ by the author and was composed within the given timelimit of 30 minutes, so that it is truly representative of the type of essay that a bright candidate is expected towrite for deserving the highest score of 6.

You can obviously not compose essays before you sit for your GRE on each of the 242 topics given, nor canyou memorize all the following essays and reproduce them in the test.

On the other hand, you should not just read the following essays a few times and then imagine that youwill have no difficulty in composing a similar essay in the test hall if the topic is posed to you. You shouldwrite essays of your own on at least 25 of the given topics before you sit for your GRE test. Composing anessay is a difficult art, and the examination hall should not be the place where you compose a GREessay of your own for the first time.

Hallmarks of an effective essayOne of the ways to sound smart while writing an essay is the appropriate use of what are known as

‘signpost’ words or phrases.Some of the ‘signpost’ words and phrases are: For instance, for example, however, on the other hand,

although, unlike, yet, nevertheless, since and therefore.An illustrative example may be introduced by the phrases ‘For instance’ or ‘For example’.A series of arguments may be introduced by the words ‘First’ or ‘Firstly’, ‘Second’ or ‘Secondly’, ....

‘Finally’.When you wish to contrast two ideas from each other, you can use signpost words such as ‘However’, ‘On

the other hand’, ‘Nevertheless’, ‘Yet’ and ‘Contrariwise’.Signpost words and phrases are valuable in many ways. They help the reader to follow your argument

more easily. They make the purpose of every detail, fact, story or example which you use more obvious.More important, they make your essay sound well-organized.By emphasizing the structure of your argument, the signpost words prove that you have thought through

the argument rather than rambling aimlessly from one idea to another, and that you have got a plan and youare following it intelligently.

So, look for opportunities to use these four kinds of signpost words in your essays:1. Words that show a contrast or change in idea: although, but, by contrast, despite, however, nevertheless,

on the other hand, unlike and yet2. Words that show a similarity or the continuation of an idea: also, as well, equally, in the same way,

American Education Aids - GRE Arguments - Model Essays 5

likewise, parallely, similarly, so, thus and too3. Words that show a time sequence: after, before, earlier, later, next, previous, prior to, subsequently and

then4. Words that show a cause and effect relationship: as a result, because, consequently, due to, led to,

produced, resulted in, since and therefore.

Vary the lengths of your sentencesOne sure method of making an essay dull, lifeless and monotonous is to have all its sentences of the same

length.To sound smart, you should consciously try to vary the lengths of your sentences. While some of your

sentences may bee fairly long with three or four subordinate clauses, others must be short with just five or sixwords. The changing rhythm of your writing will give your prose a snappy, intelligent tone, and will keep thereader alert and interested.

Instead of all sentences as narrative ones, you should also try to introduce, in appropriate places of youressay, an interrogative (which is in the nature of a question) and an exclamatory sentence.

If you wish to key in your essay instead of handwriting, you can use the limited word processing featuresof the typing program in the computer that will be made available to you in the test center.

This programme will have three features - cut, paste and undo.In the course of typing an essay, or after completing it, if you wish to delete a word or phrase or sentence,

you can do so by selecting it with your mouse and pressing the ‘cut’ command.If you wish to shift a word, phrase or sentence from one place of the essay to another, you can select it,

use the ‘cut’ command and then, pointing the cursor at the point where you wish to insert it, press the ‘paste’command.

The word processing software stored in the ETS computer does not have the ‘spell check’ facility.

You can adopt one of the following standard formats for your essay on ‘Analysis of an Argument’:

Format 1Para 1:The argument that (restatement of the conclusion in the given paragraph) is not entirely and logically

convincing because it is based on a few unproven assumptions.Para 2:First, the argument assumes that ...Para 3:Second, the argument does not address the question ....Para 4:Third, the argument omits ....Para 5:For all these reasons, the argument is not completely sound.Para 6:The argument might have been strengthened if only the author had ....

Format 2Para 1:State your position: Restate the argument.Para 2:Discuss the link (or the absence of it) between the conclusion and the evidence presented in support of it.Para 3:Show three deficiencies in the reasoning of the argument.Para 4:Show how each of the three deficiencies could be plugged by stating the missing assumptions.

Format 3Para 1:Restate the given argument and say that it has two/three/four flaws.Para 2:Point out the first flaw and show it could be plugged if only a particular assumption (which is missing) is

American Education Aids - GRE Arguments - Model Essays 6

made.Para 3:Point out the second flaw, and show it could be plugged only if another specific assumption (which is

missing) is made).Para 4:Point out the third flaw, and show it could be plugged only if another specific assumption (which is

missing) is made).Para 5:Summarize your essay with the statement that, because these assumptions have not been stated, the

given argument is weak.

Do not use strong words such as ‘foolish’, ‘idiotic’ or ‘mad’ to describe a conclusion in the given argument.Remember that logical flaws are deliberately built into the conclusions for you to notice and comment upon.The use of words such as ‘illogical’ and ‘unfounded’ are quite adequate to describe these logical flaws.

Wind up your essay with a concluding paragraph as in the model essays below. If you have time, you cando a little more elaboration of the final paragraph to state what specific additional information will berequired if the given conclusion is to be considered as valid. In most cases, this will be a restatement of whatyou have written already.

Remember that there is a time limit of 30 minutes for composing your essay. It is only bywriting a few essays on your own within this time limit that you can assess beforehand how manywords you will be able to write or type in the test hall. A length of 350 to 400 words would be theoptimum.

Remember that the wording of a topic that is posed to you by the computer when you sit foryour GRE may differ slightly from what is found in the ‘Pool of GRE Topics’ released byEducational Testing Service. Note the difference first and understand it before starting tocompose your essay.

(In the case of the first few model essays below, we have first indicated the types of flaws in the givenargument. You can take them as the guidance for reviewing the subsequent arguments.)

1. ‘Nature’s Way’ chain of stores(The basic flaw in the given argument is the use of vague phrases such as ‘many such residents’, ‘all-time

highs’, and ‘more members than ever’ which do not have any specific quantitative meaning. You should buildyour essay on this fundamental defect.)

Any proposal for a new commercial venture must concentrate on its financialviability, and must contain specific projections of anticipated turnover, cost ofgoods, gross profit, overhead expenses and net profit. The memorandum of the VicePresident is silent on each of these points.

Firstly, for any such venture to be viable, there ought to be a ‘break-even’turnover which the Vice President of any leading chain stores must be aware of. Hismemorandum does not contain a single specific figure, but talks generally of ‘manysuch residents’, ‘all-time highs’ and ‘more members than ever’, which do not have anynumerical certainty.

Secondly, the memorandum does not even indicate whether Plainsville is a small ora large village, or a small or a large town. It has no information about itspopulation. If the total population of Plainsville is 1000 (of whom only about 600are likely to be adults), the fact that ‘many of them’ are concerned about leadinghealthy lives may not justify the establishment of a new store of Nature’s Way there.

On the other hand, if the population of Plainsville is above 100,000, theproposal would look more rosy.

Thirdly, the fact that ‘sales of running shoes and exercise clothing inPainsville are all-time highs’ does not also strengthen the argument, because such‘all-time high’ may be just $500 in the whole year, the figure in the previous yearshaving been even less.

Fourthly, the memorandum says that the local health club, which nearly closeddown five years ago, has ‘more members than ever’. This vague figure of ‘more members

American Education Aids - GRE Arguments - Model Essays 7

than ever’ may be anything like 50 or 1000. Obviously, the proposal will have somevalidity if the number is 1000, but will have no justification at all if the numberis only 50.

Fifthly, the memorandum talks of the business that will be generated by schoolchildren, again without giving any information about their number. If there is onlyone school in Plainsville with less than 200 students, then the proposal will not bejustified. If it has a number of schools with a total enrollment of 10,000, therewill be some merit in the proposal.

Strangely, the proposal is silent about existing competition. Even if all thevague phrases in the memorandum refer to fairly appreciable numbers, the existence ofsimilar stores already in Plainsville with their own loyal and satisfied clientelemight render the new store of Nature’s Way non-viable.

The memorandum makes the bland assertion that ‘our next new store’ must be builtin Plainsville. Before making this categorical statement, the vice president musthave discussed the merits of other possible alternative locations for the next newstore, and then made out a convincing case why Plainsville is a better candidate thanall of them.

Thus, the proposal would have validity if he had quantified all the vague phraseswhich he has used, had made specific financial projections, discussed existingcompetition and also compared the merits of Plainsville with those of other possiblelocations for the next store.

Therefore, as it stands, the memorandum cannot be of any help to the managementof Nature’s Way to come to an informed decision.

(Note how almost all bits of information contained in the given argument have been touched upon and analyzed in thisessay.)

2. Deerhaven Acres & Brookville Community(The major flaw in the given argument is that it presumes that, just because Event B followed Event A,

Event B was caused by Event A. It does not consider the possibility that there could be other more validreasons for the occurrence of Event B. You should develop your essay on this flaw, and think of as many otherreasons as you can. You can also argue that what proved true in the case of Brookville may not prove true inthe case of Deerhaven.)

The committee’s argument is based on the assumption that it was only the new setof restrictions on landscaping and coloring of home exteriors that contributed to thetripling of property values in Brookville in the last seven years.

The argument is weak because it does not consider other possible reasons for suchincrease.

For example, the setting up of a new big industrial unit in Brookville in theintervening period which created hundreds of new employment opportunities, could havebeen the real reason for the increase in its property values.

A second possibility is that a new university or a major government laboratoryhas been set up in Brookville, and this had pushed up the property prices in thatlocality.

A third possibility is that property values in Brookville seven years back hadbeen much lower than in neighboring counties because of the existence of a highlypolluting industry there. The industry may have been closed down during the lastseven years resulting in these values getting tripled.

A fourth possibility is that, in spite of the stated increase, the propertyvalues in Brookville are still lower than those in Deerhaven, and people fromDeerhaven are themselves moving into Brookville. If so, mere changes in landscapingand wall coloring are not likely to contribute to an increase in property values inDeerhaven.

Another possibility is that a major non-polluting industry in Deerhaven hasclosed down recently, throwing a large number of people out of employment. If so, theproperty values in Deerhaven will crumble, and will not increase just because ofchanges in landscaping and similar rules.

Above all, common sense dictates that the mere changing of color schemes ofexteriors of buildings or the landscaping of the open yards do not triple property

American Education Aids - GRE Arguments - Model Essays 8

values in a large area.Therefore, the argument in the committee’s letter is least convincing as it

stands. To buttress the proposal, the committee will have to conclusively establishthat no other reason except what they have stated in their letter had triggered theincrease in Brookville’s property values, and that no other adverse factor ispresently pulling down the property values in Deerhaven.

3. Megalopolis’s law firms(This argument does not consider any alternative reason for the stated result, uses the vague phrase

increasing number without quantifying it, and incorporates a contradiction in the last two sentences. Youshould develop your essay highlighting these points.)

From the fact that the number of law school graduates who went to work for large,corporate firms declined by 15 percent in the last three years, and an ‘increasingnumber’ of law school graduates took jobs at small, general practice firms, theauthor comes to the conclusion that law school graduates are ‘choosing’ to work forthe smaller firms ‘in preference to’ large firms.

This conclusion is questionable because it does not consider the possibility thatall these students too had applied for jobs in large firms but could not beaccommodated by the latter because of a general decline in their type of business;and had no choice but to join the smaller firms.

The second flaw in the argument arises out of the non-quantification of thephrase ‘increasing number’. If the number of law school graduates joining large firmshad ‘declined’ by a mere 30 from 200 to 170 in the last three years, and the numberwho had joined small, general practice firms during this period had ‘increased’ by amere 10 from 20 to just 30, one can still conclude, contrary to what the author says,that large firms continue to be the preference of a large number of law schoolgraduates.

Thirdly, as the reason for the unsubstantiated preference of law graduates forsmall firms, the author cites the opinion of ‘first year’ students that earning ahigh salary was less important to them than job satisfaction. There are two problemswith this argument: (i) Can the opinion of ‘first year’ students be extrapolated asrepresenting the opinion of a majority of law school ‘graduates’? (ii) Is therejustification for the implied assumption that ‘working in large firms’ isincompatible with ‘job satisfaction’?

It is quite likely that students who are idealistic in their first year in lawcollege become more practical or mercenary when they are about to graduate. They maythen consider that it is working in big firms that will give them greater jobsatisfaction because of the wider opportunities that they will get there to handledifferent types of legal problems.

Finally, the last sentence in the article does not follow logically from theprevious assertions and, in fact, contradicts them. If law students are not lured by‘high salary’ but prefer ‘job satisfaction’, why does the author argue that largefirms must offer ‘more benefits and incentives’ in order to attract them? And wherehas he cited evidence that one need work for shorter hours in smaller firms than inlarger firms?

For the foregoing reasons, the argument advanced in the article does not havemuch force.

4. Adams Realty & Fitch Realty(‘Realty’ is the business of acting as a broker between a seller and a buyer in real estate transactions. The

flaws in this argument are: judging the relative competence of two firms on the basis of just two or threeparameters, and comparing a 10-year-old event with a 1-year-old event to judge the relative competence of thetwo firms. You should build your essay around these flaws.)

It is on highly specious grounds that the author of this notice wants the readersto believe that Adams Realty is more efficient than Fitch Realty.

Firstly, the statistics given in the first two sentences merely show that Adamshas more human resources than Fitch. This does not, by itself, warrant the conclusionthat Adams is ‘clearly’ superior.

American Education Aids - GRE Arguments - Model Essays 9

In fact, the given statistics may themselves be interpreted to prove thecontrary: If Fitch has 50% as much revenue as Adams has, but with only 5/8 as manyreal estate agents as the latter has (and they too working only part-time), it isFitch which should be considered as the more efficient between the two!

Secondly, the efficiency of a real estate agent (from the point of view of theseller) must be assessed on the basis of how much higher price than the market valueof the property that he is able to get for his client. It is quite probable that theaverage ‘market value’ of the properties sold by Fitch was only $125,000 but thisfirm was able to get an average price of $144,000 for its clients; similarly, theaverage ‘market value’ of the properties sold by Adams might have really been$175,000, but it was able to get an average price of only $168,000 for its clients.If this be the real case, who between the two can be named more efficient? ObviouslyFitch, and not Adams!

Thirdly, the author compares his experience in employing Fitch ten years backwith his experience with using Adams last year, and recommends the latter to otherprospective property sellers. This argument is highly questionable because he doesnot account for the possibility that the real estate market was bearish ten yearsback with very few buyers in the market, but had turned highly bullish last year. Italso does not consider the possibility that the average time taken by Fitch to sellproperties last year was less than one month, a better record than that of Adams.

For the above reasons, the recommendation that Adams must be used in preferenceto Fitch both for quickness of the transaction and for getting a higher price is notlogically established by the stated reasons, and requires much more relevant andreliable data before it can be acted upon by other prospective property sellers.

5. Becton Pharmaceuticals & Perkins Pharmaceuticals(You should first identify which are the ‘facts’ or the ‘opinions of others’ and which are the ‘conclusions of

the author’, and then write a critique on only the conclusions of the author.The probable rise in the number of arthritis patients from 40 million to 60 million is a ‘general assumption’, while

‘pharmaceutical companies that produce drugs for the treatment of arthritis should be very profitable’ is a conclusion of theauthor. Similarly, ‘Becton Pharmaceuticals, which makes Xenon, will be the most profitable pharmaceutical company’ is thebelief of ‘many analysts’, while the author’s conclusion is that ‘Perkins Pharmaceuticals is more likely to be the winner’. Youshould write a critique of only these two conclusions of the author.)

While the increase in the number of arthritis patients in the next twenty yearsis likely to result in the increase in the sales of drugs for arthritis, it does notwarrant the conclusion that pharmaceutical companies which produce drugs forarthritis should therefore be ‘very profitable’.

Firstly, a higher turnover does not always result in a higher profit, since theprice of raw materials may go up without the company being able to pass on the highercost arising therefrom to the consumer.

Secondly, if a large number of pharmaceutical firms start manufacturing Xenonafter its patent expires in the next three years, the market is likely to get highlyfragmented, and the resulting price war may result in a situation of low profit forthe entire industry manufacturing this drug.

Moreover, the author does not envisage the possibility that non-drug-based curesfor arthritis (such as laser treatment) may be developed within the next twenty yearsresulting in lower consumption of drugs for arthritis.

Seen from this angle, the first assertion of the author is not as sound as itprima facie appears to be.

The second conclusion of the author that Perkins Pharmaceuticals will be ‘moreprofitable’ than Becton Pharmaceuticals is based on the research finding that sevenout of ten patients suffering from the ‘most extreme cases of arthritis’ prefer Xylanto Xenon. This conclusion will be valid only if most of the additional 20 millionarthritis patients of the next twenty years are afflicted by the ‘most extreme casesof arthritis’. If, on the other hand, most of these new incidences of arthritis areof less serious varieties which can be cured through the administration of milderdrugs, it is Xenon (or its equivalents) which will have a larger market. Consequentlythe author’s prognosis will fail.

American Education Aids - GRE Arguments - Model Essays 10

Moreover, his statement that other companies will produce cheaper versions ofXenon than Becton can is also not logically sound, because a depreciated plant asthat of Becton can most probably produce the drug much cheaper than any new plantthat starts manufacturing the same drug.

For the foregoing reasons, it is not possible to concur with the forecasts of theauthor of this article merely on the basis of the evidence and the reasons cited byhim.

6. Monroe’s new Jazz Club (You should first sift between the ‘facts’ and the ‘conclusions’ contained in the note. The facts are:

Currently, the nearest jazz club is 65 miles away from Monroe; over 100,000 people attended Monroe’s jazzfestival last summer; several well-known jazz musicians live in Monroe; the highest rated radio program inMonroe is ‘Jazz Nightly’; and the typical jazz fan spends close to $1,000 per year on jazz entertainment.

The ‘conclusions’ in the loan application are: A jazz club in Monroe would be a tremendously popularenterprise; the proposed club - C Note - would have the local market all to itself; jazz is extremely popular inMonroe; and it is clear that the C Note cannot help but make money.

Note that the given note is contained in an application for a ‘small business loan’ by a group of developers.You have to critically assess whether the conclusions follow from the stated facts.)In spite of the stated facts, namely, that several well-known jazz musicians live

in Monroe, that the highest rated radio program in Monroe is ‘Jazz Nightly’, and thatover 100,000 people attended Monroe’s jazz festival last year, the note states thatthere is no jazz club presently in Monroe, and that the nearest one is 65 miles away.

This means that, in spite of these favorable factors, either nobody had venturedin the past to start a jazz club in this city, or the clubs started here earlier hadclosed down.

There must have been some valid reasons for this phenomenon, but the note doesnot even attempt to probe into such reasons. If no such club had been started in thepast, the applicants for the loan would have to explain why such an obvious businessproposition had not occurred to anyone else earlier. If jazz clubs that had beenstarted earlier had closed down, the applicants must have given reasons for suchclosures, and should have indicated how they propose to avoid the mistakes, if any,committed by earlier promoters.

Secondly, it must be noted that the population size of Monroe is not mentionedanywhere in the note. It is quite probable that a majority of those who attended lastyear’s Monroe Jazz Festival had come from outside. Therefore, this figure of 100,000cannot be taken to be indicative of the size of the local population.

Moreover, it is intriguing why several well-known jazz musicians should be livingin Monroe when it does not have a jazz club and the nearest club is as far as 65miles away. One possible explanation is that Monroe is a small, prestigious,exclusive and upmarket community of very rich persons. If this is true, such a smallpopulation will not be able to economically sustain a viable jazz club.

Even assuming that there is a good potential for starting a jazz club in Monroe,the credentials of the applicants for the loan to run it successfully are notestablished. The preamble to the note says that the proposal has been submitted by ‘agroup of developers’. The competence of a set of property developers to run a musicclub successfully is open to question. The note does not indicate how they propose toovercome their lack of relevant experience to run such a club.

Even if all the data stated in the note are true, it only establishes there ispotential for establishing a jazz club in Monroe. The statements that it would proveto be “a tremendously profitable enterprise”, and “it cannot help but make money” arenot substantiated with any figures involving investment, revenue or expenditure. Itis quite likely that, in order to exploit the full business potential, a much largerinvestment than ‘a small business loan’ will be needed. If this is true, thepromoters may not achieve the full potential with the proposed limited investment.

It is stated that the average annual expenditure of $1000 on jazz entertainmentis based on a ‘nationwide’ survey. In the absence of any information about theeconomic condition of Monroe residents, it is not possible to assume that theiraverage expenditure on jazz entertainment will amount to the same figure.

American Education Aids - GRE Arguments - Model Essays 11

Even assuming that, to start with, the local jazz market may belong fully to CNote, its success may trigger off competition quickly. If new clubs emerge withbetter attractions, more efficient management and more aggressive marketing, C Notemay suffer financially and may not continue to remain profitable.

In my view, the application for a loan by the intending promoters of the club hasto be carefully examined in the light of the above-mentioned reservations.

(Note how this essay picks logical holes in a seemingly sound proposition.)

7. Clearview mayoral election(The ‘facts’ contained in the letter to the newspaper are that Frank Braun is a member of the present

Clearview town council; the number of factories in Clearview has doubled during the past year’ air pollutionlevels have increased; and the local hospital has treated 25% more patients with respiratory illnesses.

The ‘conclusions’ drawn by the writer are that the current members of the town council are not protectingthe environment; residents of Clearview should vote for Ann Green who is a member of the Good EarthCoalition, and that, if she is elected, the environmental problems in Clearview will certainly be solved.

It is these ‘conclusions’ which you must examine critically.)In advocating that residents of Clearview should vote for Ann Green in the next

Mayoral election for the reason she will ‘certainly solve the environmental problemsin the town’, the writer is presuming that the most important problem of the town isenvironmental, and that the decision of the voters should be based on this singleissue alone. But he does not give sufficient data to validate this presumption.

The statement that ‘air pollution levels have increased in the past year’ doesnot, by itself, imply that they have reached anywhere near intolerable limits. If thetolerable level of air pollution is, say, 500 particulates in a cubic meter of air,and the level in Clearview had increased in the past one year from 10 particulates to15 particulates in a cubic meter of air, there is no need to feel jittery about theincrease. If such be the case, one’s voting preference should clearly not bedetermined by such an insignificant increase in the pollution level.

Secondly, the statement that ‘the local hospital has treated 25% more patientswith respiratory illnesses in the last year’ does not, by itself, establish that theproblem has become acute. If, in a population of 100,000, the number of patientstreated for respiratory illness had increased from 100 to 125, it does not constitutea valid reason for ringing the alarm bell. So, unless the writer cites exact figuresof such patients vis-a-vis the total population of the town, it is not possible toagree with his conclusion that ‘the current members are not protecting theenvironment’.

The refrain of the writer is that, for the Mayor’s post, Ann Green must bepreferred to Frank Braun because the latter is a member of the present council. Evenassuming that the present council had taken certain decisions which had adverselyaffected environmental conditions in Clearview, Frank Braun cannot be heldresponsible unless it is shown that he had actively supported those decisions. It isquite likely that Frank Braun had stoutly opposed those measures but he was overruledby the majority of the other members in the council. In the absence of anyinformation on this aspect, one can even presume that Frank Braun is a more ardentsupporter of environment than Ann Green. In that case, the writer’s advocacy of AnnGreen’s candidature vis-a-vis Frank Braun would be misplaced.

The writer makes it appear that, in selecting the new mayor, the predominantconsideration should be his/her commitment to environment, but he does not give anyunassailable justification for his view. If, on the contrary, the most seriousproblem of Clearview is lack of employment for its residents, then a person who laysemphasis on greater economic activity and who takes steps to accelerate industrialgrowth in the town would prove to be a better Mayor.

Finally, the assertion in the last sentence, “If we elect Ann Green, theenvironmental problems in Clearview will certainly be solved” is also questionablebecause the majority of newly elected members of the town’s council may continue tobe the opponents of the Good Earth Coalition.

For the foregoing reasons, the writer of the letter has not made out a convincingcase for voting for Ann Green in the next mayoral election.

American Education Aids - GRE Arguments - Model Essays 12

8. Omni Inc and Mesa Foods(The facts in this case are: “Mesa Foods is a manufacturer of snack foods that currently markets its

products within a relatively small region of the country; Mesa enjoyed a 20% increase in profits last year; itsbest-selling product, Diabolique Salsa, has had increased sales over each of the past three years; Omni Inc. isinterested in reaching 14-to-25 year olds”.

The opinions or conclusions of the writer of the note are: “Mesa Foods has strong growth potential; OmniInc should buy Mesa Foods, and concentrate in particular on marketing Diabolique Salsa throughout thecountry”.

It is the latter which we should critically examine in the light of the facts stated first.)The proposal in this case is that a company, Mesa Foods, which has been

concentrating on marketing its snack foods within a small area, must be purchased bya nationwide company, Omni Inc.. The reason cited is that the main snack foodproduced by the former, namely Diabolique Salsa, can be marketed throughout thenation successfully.

In order to buttress his recommendation, the author says that Mesa Foods enjoyeda 20% increase in profit last year. This statement by itself is not forceful unlessthe increase in profit is quantified and not given in terms of a vague percentage. Ifthe profit had increased merely from $1000 to $1200, the argument would not have muchmerit. But, if the profits in the two successive years had been 1 million dollars and1.2 million dollars, the recommendation would be quite convincing. The absence of theexact figures of profits in the two successive years renders the recommendationrather hollow.

Equally vague is the statement that Diabolique Salsa, Mesa Food’s best-sellingproduct, has had ‘increased sales’ over each of the past three years. Therecommendation would be valid if the increases had been in the region of 200%, 500%and 800%, but would have no force if the increases had been a uniform 10%, that tooon an already small base. Thus, this statement also does not add strength to therecommendation because of its vagueness.

Another questionable assumption in the given note is that a snack food that hasbeen found to be popular with youths in a limited geographical region of the countrywill be equally popular with youths in the whole country. If the population of theregion in which Diabolique Salsa is presently marketed is predominantly of aparticular ethnic group, say Mexicans, their favorite food may not enjoy the samepopularity with the youths of the whole country, most of whom belong to various otherethnic groups.

For the foregoing reasons, the recommendation cannot be acted upon straightawaywithout further detailed examination of the issues pointed out above.

9. Omega & Alpha Universities(The main flaw in this recommendation is the attribution of a particular result to a specific single cause

without examining other possible causes. You should build your essay on this theme.)In this note, the dean of Omega University makes an unfounded insinuation that,

in order to gain popularity with their students and thereby to get a better ratingfrom them, professors in the university are deliberately assigning higher grades totheir students than what the latter deserve. According to him, this is the reason whyOmega graduates are not as successful in securing jobs as are graduates from thenearby Alpha University.

On the basis of this reasoning, the dean recommends that Omega should terminatethe practice of evaluation of the professors by the students.

The major flaw in his argument is that he attributes the increase in the overallstudent grade averages by 30% during the past 15 years solely to the alleged wish ofthe professors to placate their students in order to get a higher rating from them.He does not examine the possibility that such increase might have been due tostricter norms in the admission of students during the same period, because of whichOmega is now having relatively brighter students than what it used to enroll.

On the contrary, the increase in the student grade average could also have beenbecause Omega had dispensed with the services of inefficient professors on the basis

American Education Aids - GRE Arguments - Model Essays 13

of the students’ evaluation about them, and has now recruited more competentprofessors who teach better.

A second inference by the dean is that the reason why Omega alumni are not assuccessful in securing jobs as Alpha alumni are is that the prospective employersbelieve that the student grade averages of the former are inflated. This also suffersfrom the same flaw of not examining other possible reasons for the state of affairs.It is quite likely that the courses offered by Alpha have been periodically updatedto be more in tune with the needs of the job market while Omega has been sticking toits conventional courses which have become irrelevant in the market place.

Moreover, it is also possible that the placement office of Alpha has morecompetent personnel than that of Omega. If this is the case, Omega graduates will beable to get a better deal if this university also strengthens its placement officeinstead of going back on the student assessment scheme.

Paradoxically, the dean’s note does not state that Alpha University does notfollow the policy of student evaluation of faculty. If the same practice is beingfollowed in Alpha also, the bottom is taken out of the dean’s argument.

Assuming that the dean’s reasoning is valid and the student evaluation of facultyis dispensed with, the immediate result will be that the grade average of Omegagraduates will come down by 30%. At least in the short term, until the employersstart appreciating the fact that the grade averages awarded by Omega truly reflectthe merit of the students, Omega graduates with lower grade averages will be evenworse off in the job market than they are at present.

For the above reasons, the recommendation of the dean should not be implementedstraightaway, and the authorities must probe into the reasons much more exhaustively.

10. Batavia’s milk prices(The ‘facts’ in this case are: The number of dairy farms throughout Batavia is 25% greater than it was 10

years ago. During the same 10 years, the price of milk in the local Excello Food Market has increased from$1.50 per gallon to over $3.00 per gallon.

The other statements in the narration are ‘opinions’ of the writer. It is these opinions which you mustexamine critically.

The flaws in his reasoning are equating an increase in the ‘number of dairy farms’ to an increase in the‘extent of acreage’ devoted to dairy farming; presuming that an increase in the retail price of a product resultsin an increase in the profit of its producers; and that government regulation should protect the consumers’interests as against the producers’ interests.)

The first inference of the writer of the letter is that the increase in thenumber of dairy farms in the whole of Batavia must have necessarily resulted in anincreased supply of milk and consequent reduction in its price in the local retailshop.

This inference is questionable because the increase in the ‘number of dairyfarms’ need not have resulted in a simultaneous increase in the ‘farm area devoted tomilk production’. On the contrary, it is quite possible that large dairy farms whichhad existed ten years back have now got fragmented into a number of smaller farms,even while a considerable extent of land which was earmarked for dairy farming hassince been diverted for other uses. If this is the case, there would have been areduction, and not an increase, in milk production in Batavia despite an increase inthe number of dairy farms.

Citing the increase in the price of milk in the local retail shop during the pastdecade, the writer arrives at the further conclusion that dairy farmers are making‘excessive profits’. Obviously he presumes that the farmers have not suffered aconsiderable increase in the cost of production of milk during the last decade. Ifhis presumption is wrong, and the cost of inputs for the dairy farmer has gone upeven more than the increase in the retail price of milk, then the dairy farmerswould, in fact, have been making even less profit now than they did ten years back.

The writer also presumes that the entire increase in the retail price of milk isbeing appropriated by the dairy farmer. On the contrary, it is quite probable thatthe increase in the retail price of milk is mainly attributable to a substantialincrease in its cost of treatment, preservation, transport and distribution, and the

American Education Aids - GRE Arguments - Model Essays 14

dairy farmer is getting the same price now as, or even less price now than, what hewas getting ten years back. If this is the true state of affairs, the writer’scomplaint about ‘excessive profits’ being made by dairy farmers would again bebaseless.

Finally, it is also likely that, while the number of farms in the whole ofBatavia has increased, the number (and the area devoted to dairy farming) hasdecreased in the local area in which Excello Food Market is situated, because ofwhich milk has to be transported a much longer distance to reach the local market.This could be the major cause for increase in the retail price of milk in the localretail outlet.

In the last sentence, the writer says that government regulation of retail priceis necessary to ensure both lower prices and an adequate supply of milk forconsumers. Ironically there is an inherent contradiction in this conclusion: if thegovernment regulation results in non-remunerative price for the dairy farmers, theymay well discontinue the production of milk and divert their land for other moreremunerative enterprises. If this happens, the supply of milk to consumers will, infact, lessen and not increase. Consequently, the price of milk will increase further.

All the doubts pointed out above must be satisfactorily clarified before we canaccept the conclusions of the writer of the letter.

11. West Egg’s garbage disposal(The weakness in this argument is the vagueness in the meaning of the phrases ‘twice as much

aluminum and paper’, ‘charges will double’, and ‘respondents to a recent survey’. You should build your essayon this theme.)

The first reason cited by the Mayor for his conclusion (that the available spacein the landfill should last considerably longer than the predicted five years) isthat town residents have been recycling ‘twice as much aluminum and paper as they didin previous years’.

This is a vague statement without specific quantification. If the averagepercentage of aluminum and paper in garbage is more than 40% by volume, the Mayor’sargument will have considerable force. On the contrary, if it is less than 1% byvolume, the fact that residents nowadays prefer to recycle them instead of throwingthem into garbage will not make an appreciable difference to the predicted life ofthe landfill.

Secondly, the Mayor predicts that, since garbage pickup charges will double nextmonth, the amount of materials recycled should further increase. This statement againdoes not give the specific figure of pickup charges. If the charges are gettingdoubled from $100 a month to $200, the Mayor’s prognosis may probably come true. Ifthe charges are only getting doubled from $5 to $10, it may not be sufficientincentive for people to resort to more recycling.

Another reason cited by the Mayor for his conclusion is that over 90% of therespondents to a recent survey said that they would do more recycling in future. Notethat he does not say that 90% of the ‘residents’, or even that 90% of ‘those who weresurveyed’ expressed this opinion, but that 90% of ‘those who responded to the survey’said so. If only 10% of the residents had been surveyed, and 90% of them did notrespond at all to the survey, then the opinion cited by the Mayor would be that ofonly 1% of the total number of residents. This, obviously, cannot form a validjustification for his conclusion.

Lastly, even assuming that all the reasons cited by the Mayor are valid, he doesnot furnish evidence that the consultants had not considered these possibilitiesbefore they predicted the life of the landfill as five years. If the consultantsthemselves had already factored these possibilities into their prediction, then againthe Mayor’s conclusion would be wrong.

Before the Mayor’s prediction is accepted and acted upon, the Council should getclarifications on each of the issues mentioned above.

American Education Aids - GRE Arguments - Model Essays 15

12. Alta Manufacturing & Panoply Industries(The basic defect in this case is the attribution of just one cause for the stated result without considering

other possible causes. You must build your essay on this theme.)The basic flaw in the reasoning of the Vice President is that he assumes that

there can be no other reason except the longer duration of the work shifts for thegreater number of accidents in Alta Manufacturing than in Panoply Industries.

On the other hand, it is quite likely that, even as types of industries, Alta ismore prone to accidents than Panoply. If Alta is making overhead cranes and Panoplyis making computer chips, the former will have much more accident potential than thelatter, irrespective of the lengths of the shift times. In order to agree with thegiven conclusion, the Vice President should first convince us that Alta and Panoplyare comparable industries from the point of view of accident proneness.

Even if it be true that Alta and Panoply are comparable industries in thisregard, the greater number of accidents in Alta may be because its machines are mucholder and do not have as many built-in safety gadgets as the newer ones in Panoply.If this is the case, the solution to the problem lies in modernizing Alta’s machinesand not in reducing the working hours of the employees.

Moreover, it is possible that Panoply puts its workers through safetyconsciousness exercises more frequently and much more effectively than Alta does, orthat the management of Panoply enforces safety precautions much more strictly thanAlta. If this is the real reason behind the observed fact, then again the reductionof working hours for workers is not the solution.

Note that the vice-president compares the actual number of accidents in the twofactories, and does not relate them to the number of workers. If Alta has 2000workers and Panoply has only 100, and the number of accidents in the two are 13 and10 respectively, it is Alta which should be considered a safer factory than Panoply!

It is apparent from the note that there are as many as three shifts in Alta,which means that the working hours per shift last a maximum of only eight hours. Theworkers have the remaining 16 hours to themselves. Since an average adult needs tosleep for only 6 to 7 hours a day, the Vice President has not made out a case thatthe present working hours in Alta leads to sleep deprivation of its workers.

Even assuming that his argument is valid, how can one be sure that the workerswill use the one hour additionally made available to them for sleeping and not forother activities? If they continue to sleep for the same number of hours as they donow, then the reduction of one hour in their working time will not contribute to lessnumber of accidents. On the other hand, working for one hour less per day, they willbe less productive, nullifying the other prediction of the Vice President that hisrecommendation will result in increased productivity.

In my view, the recommendation of the Vice President cannot be acted upon by themanagement of Alto unless the concerns mentioned above are satisfactorily addressedfirst.

13. ‘Squirrel Power’ novel by Peter WoodThe first reason that the chief executive cites in support of his recommendation

to acquire the video game rights of the yet-to-be published children’s book ‘SquirrelPower’ by Peter Wood is that this author’s last three books were best sellers.

This by itself is no guarantee that the next book by Wood will also be a bestseller. If the new book proves to be much less popular than the previous ones, thesubstantial investment suggested for buying its rights even before it is actuallypublished may prove to be a costly gamble which the company can ill afford. Note thatthe CEO himself has referred to the economic constraints that the purchase can causeto the company.

Secondly, he attempts to reinforce his first argument by pointing out that themovie based on Wood’s first book was highly profitable. There is no informationwhether a movie was released based on his second book also and, if so, whether it wasalso equally profitable or had been a flop.

Another question that needs to be answered is: can the fact that a movie based onan author’s book was successful be taken as the guarantee that a video game based on

American Education Aids - GRE Arguments - Model Essays 16

another book by him will be equally successful? To add force to his recommendation,the CEO must have cited specific precedents in which both movies and video gamesbased on a number of popular books had been equally successful.

Moreover, the argument in the last sentence in the note is also open to question.How does the CEO presume that the characters and the story line of the new book willbe ‘popular’ when the book is yet to be published?

Finally, he has not made out a convincing case with actual figures why the costof developing a storyline and characters for a new video game in-house will begreater than the admittedly substantial cost of acquiring the rights to an yet-to-bepublished book.

The CEO must address all the doubts raised above satisfactorily in asupplementary note before his management can act upon his recommendation.

Perhaps, a better option before the company will be to purchase the rights tocreate a video game based on the first book of Wood which had proved to be both abestseller and a popular movie story, because such rights of an earlier book may beavailable at a substantially lesser price.

14. Green Thumb Gardening CenterThe first reason cited by the owner for his recommendation is that a ‘national

survey’ conducted last month indicated that ‘many’ consumers were dissatisfied withthe quality of fresh vegetables available in supermarkets.

I find two problems with this argument. The first is that ‘many consumers’ is toovague a phrase, and may denote any figure between 5% and 75% of the total number ofconsumers. If the dissatisfaction expressed is that of only 5% of the consumers, theargument of the owner will obviously have no force. So, his first obligation is toquantify this vague figure.

The second problem is: can the finding in the ‘national survey’ be appliedwithout relevant adjustments to the situation in a ‘suburban town’? If the suburbantown is located in the midst of a farming area, then the vegetables available in thelocal supermarket are likely to be much more fresh than those available in thesupermarkets in big cities which are far away from production centers.

It could even be the case that, in this suburban town, consumers do not dependupon supermarkets for their vegetables but buy them from smaller shops run by thefarmers themselves. In this case also, the recommendation of the owner will have lessvalidity.

Another reason cited in the note is that the gardening magazine ‘Great Gardens’has sold out in the local news stand three months in a row. It does not say how manycopies of this magazine had in fact been sold. If the number of gardeners in thesuburban town is 500 and the number of copies ‘sold out in the local news stand’ isonly 25, the recommendation of the owner will hardly be justified.

Additionally, he does not also clarify whether ‘Great Gardens’ concentrates onflower gardens or vegetable gardens. If the contents of this magazine relate mostlyto flower gardening and not to vegetable growing, its popularity among the localgardeners will not justify the stocking of many more vegetable seeds by the owner’sgardening center.

Finally, advisability of stocking ‘a large variety’ of vegetable seeds is alsoopen to question. Will the climate of the place allow all these many varieties tothrive and grow in it?

The owner of the Green Thumb Gardening Center will have to ponder over all thesepoints before taking the step proposed by him.

15. Investments in Old Dairy’s stocksThere are a number of problems with the first sentence in the newsletter.How many persons were actually approached during the survey, and what percent of

the total population did they constitute?And what percentage of those who were approached during the survey did actually

respond to it?The newsletter is silent on each one of these relevant points.

American Education Aids - GRE Arguments - Model Essays 17

If those approached constituted 80% of the total population, if 90% of them hadresponded to the survey, and over 80% of these had said that they would like toreduce their fat and cholesterol intake, the recommendation will have considerableforce.

On the contrary, if only 5% of the total population had been approached, and only10% of them responded to the questionnaire, then the opinion cited in the newsletteris that of only 0.4% of the total population. If this be the case, therecommendation, based on this survey alone, will obviously have no validity.

Without specific information on these two important aspects, it is unsafe toagree with the author of this newsletter.

Moreover, the age composition of those surveyed is not mentioned either. The factthat, say, a majority of over 60-year-olds preferred non-fat food will not mean thatthe preference of the overall population is also the same. This point must also beclarified before the recommendation can be acted upon.

The phrase ‘many food stores’ in the first sentence is also delightfully vague,and may represent any number from 10 to 500. The validity of the recommendation willonly be proportionate to the actual value of this undisclosed number.

Even if we concede the arguments in the first sentence, the author has not citedany basis for his assumption that the management of Old Dairy Industries are notaware of these problems and are not planning to change their product mix to suit thedifferent demands in the emerging market. If they are already doing so, it is quitelikely that the company will become even more profitable than it is now, and thevalue of its stocks will consequently appreciate.

Thus, the newsletter has not made out a convincing case to justify therecommendation contained in the last sentence.

Surprisingly, the authors of this newsletter have not realized the inherentcontradiction between the two recommendations they have made: that existingstockholders of Old Dairy should sell their stocks and that other investors shouldnot buy these stocks. How can anyone sell his stocks if noone else is prepared to buythem?

16. Impecunia’s lottery proposal like Lucria’s(The first three sentences in the letter are facts, and only the last sentence incorporates the conclusion of the

writer. And your critique should concentrate on the last sentence.)The assertion of the writer of the letter that the lottery proposed by him ‘would

doubtless be successful’ is based on the results of a survey conducted in the‘capital city’ of Impecunia, which showed that its citizens already spend an averageof $50 per person per year on gambling.

The first problem with this assertion is that it tries to extrapolate thebehavior of the people of the capital city to the people of the state as a whole. Ifthe people in the rural areas and in small towns of Impecunia are not used togambling of this order, the prediction of the writer will obviously fail.

Secondly, the letter does not mention the percentage of the population of thecapital city to the total population of the state of Impecunia. If this is as much as95%, then the results of the survey can be acted upon. On the contrary, if thecapital city’s population forms only 5% of the national population, then this surveywill not be of much use in predicting the fate of the proposed lottery.

The third problem with the prediction is that it presumes that, once the newlottery is introduced, the citizens will start subscribing to it in preference towhatever else they are gambling on at present, and that the average expenditure of$50 per capita on gambling will accrue to the new lottery. The writer does not citeany valid reason for this presumption. If people have all along been used to gamblingmainly on horse-racing or on speculation in the stock market, they are not likely togive these habits and switch over to speculating in the new lottery.

Nor is it likely that the citizens of Impecunia will increase the average amountof $50 that they are presently spending on gambling to additionally invest in theproposed lottery.

In justification of his proposal, the author gives comparative information on

American Education Aids - GRE Arguments - Model Essays 18

expenditure per pupil and number of persons covered by the public health system inImpecunia and Lucria, the neighboring state which introduced a similar lotterysuccessfully last year in order to finance these two specific purposes. It is quitelikely that Impecunia does not need to incur additional expenses on these two countsbecause of its different demographic composition, such as its populationpredominantly consisting of people in the age group 20 to 50.

For the foregoing reasons, the assertion of the writer of the letter cannotstraightaway be acted upon, and a further careful examination of the issues raisedabove is necessary.

17. Walnut Grove’s garbage disposal(The main flaw in this case is the vagueness about what percent of the total residents of Walnut Grove

were covered by the survey, and what percent among them actually responded to the survey. Moreover, thereis no information about how the questionnaire was worded. You should build your essay around these points.)

The conclusion in the last sentence that EZ provides ‘exceptional service’ inrespect of garbage disposal is based on the fact that 80% of the respondents to lastyear’s town survey said that they were ‘satisfied’ with EZ’s performance.

I find two problems with this conclusion. It does not say what percent of thehouseholders of Walnut Grove were covered by the survey, and what percent of thosewho were given the questionnaire actually responded to it. If the survey covered asample of 10%, and only 20% of them actually responded to it, then the writer’sconclusion will be based on the opinion of only 1.6% of the total householders, andwould have no merit.

On the contrary, if the survey covered the entire 100% householders, and 90% ofthem had responded to it, then the opinion recorded would be that of nearly 72% ofthe householders, and could be taken as a representative opinion.

Even in the latter case, it would be illogical to conclude that the response‘satisfied’ even by such a large majority can lead to the conclusion that theperformance of EZ is ‘exceptional’. The note does not indicate the options given tothe householders on how they should grade the performance of EZ. If the options hadbeen ‘Exceptional’, ‘Excellent’; ‘Good’ and ‘Satisfactory’, then the result of thesurvey would clearly go against EZ. If the options had been ‘Good’; ‘Satisfactory’;‘Unsatisfactory’ and ‘Poor’, there could be some merit in the writer’s argument thatthe people are not against EZ. In the absence of any information in this regard, thewriter’s recommendation cannot be acted upon straightaway.

The other problem with the assertion of the writer is that he presumes that ABC,if offered the contract, will not also agree to clear the garbage twice a week, andthat it will also not decide to increase the strength of its fleet for this purpose.

The author’s presumption is that the five trucks ordered by EZ will augment itsfleet. On the contrary, it is quite possible that these have been ordered by EZ inreplacement of the aged trucks it is presently using, and not to augment its fleet.If so, there will be no addition to the number of vehicles utilized by EZ forclearing the garbage.

Therefore, if the town council is negotiating with ABC to agree to clear thegarbage twice a week as EZ is doing at present but at a lesser fee of $2000, itsproposal should be considered to be in the interests of the city and must besupported.

18. Prunty County’s speed limitThe editorial advocates the restoration of the speed limit in Prunty County to 55

miles for the reason that the earlier reduction of the speed limit to 45 miles hasnot resulted in an appreciable reduction in accident rate.

I find this advice grossly illogical. If the reduction of the speed limit from 55miles to 45 miles has not reduced the accident rate, the logical solution is to lowerthe speed limit still further to 40 or 35 miles, and not to increase it to 55 milesagain.

It is well known that, when the road ahead looks clear, motorists all over theworld take some liberty with the prescribed speed limit. If the limit is 55 miles,

American Education Aids - GRE Arguments - Model Essays 19

they may try to accelerate their vehicle to 60 miles; if the limit is 45 miles, theymay try to run at 50 miles or so. If the motorists using Prunty County roads alsoindulge in this universal practice, it is desirable to keep the speed limit at 45miles and not increase it, so that the maximum speed attained by them remains around50.

In fact, enforcing the revised speed limit more strictly by engaging additionalpolice squads may be a cheaper solution to the problem than what is proposed in theeditorial.

Increasing lane widths and resurfacing rough roads are standard solutions totraffic problems. But the editorial does not state clearly that the lane widths inroads in Butler County after the recent widening are larger than those in PruntyCounty. It is quite likely that the lanes in Butler County were much narrower fiveyears back than those in Prunty County, and have only now been brought to thestandard of the latter. If so, there is no force in the recommendation that PruntyCounty should further increase its lane widths.

The last sentence points out, as is only logical, that the improvements made tothe roads in Butler County had resulted in a 25% decrease in its accident rates. Butit does not compare the accident rates in Butler County to those in Prunty County.If, in spite of the 25% decrease over a period of time, the accident rate in theformer is greater than that in the latter, it is Butler County which should reducethe speed limit to 45 miles per hour, and not Prunty County which should increase itto 55 miles per hour.

Lastly, the editorial does not examine whether the terrain in Prunty County willallow the widening of its roads at a reasonable cost.

Before the government of Prunty County accepts the recommendation in theeditorial, it has to get the above points examined carefully.

19. Double Rice’s concerts(The first flaw in this case is the attribution of the financial success of one concert of Double Rice this year

to just only reason - namely the engagement of a particular advertising agency to promote the concert - andnot conceding that there could be other more relevant reasons for the success. The second flaw is thepresumption that the good performance of the advertising agency in a single city will be repeated in the entirecountry. You should build your essay around these points.)

The recommendation in the manager’s letter is that the advertising agency Ad Libshould be hired on a nationwide scale to ensure the success of Double Rice’s futureconcerts throughout the country.

The justification he gives for his recommendation is that all the 30,000 ticketsfor its recent concert in Megalopolis, for which Ad Lib ran the advertising campaign,sold out within 12 minutes, whereas the completion of sale used to take at least aday for earlier concerts.

I find a number of problems with his recommendation.First of all, whether it takes 12 minutes or a full day for all the tickets to

get sold does not really make much of a difference. What is required is that all thetickets must get sold before the commencement of the concert.

Even assuming that this fact makes a difference, was the capacity of the stadiumin Megalopolis comparable to those in which earlier concerts of Double Rice had beenheld? If the average capacity of the stadia in earlier concerts was, say around100,000, selling that many tickets would certainly have taken a longer time thanselling 30,000 tickets. If so, the shorter time in the case of Megalopolis cannot beattributed solely to the efficiency of Ad Lib.

Secondly, it is quite likely that Megalopolis is a university town with a largepopulation of youths in the age group 14 to 25 among whom there has been a recentsurge in popularity of Double Rice, probably because one of its recent numbers hasalready been a big hit. In this case also, the quick sale of tickets for theMegalopolis concert may not be due entirely to Ad Lib’s publicity efforts.

Moreover, the manager does not give information about the admission prices forthe concert last year and this year. If the price was $10 last year and only $4 thisyear, this could be the real reason for the tickets having been sold out quickly, and

American Education Aids - GRE Arguments - Model Essays 20

no great credit is due to the advertising agency.The manager himself says that it was till about a year back that there had been

some tardiness in the sale of tickets for Double Rice’s concerts, but that itspopularity has since been increasing. The quickness of the sale of tickets inMegalopolis might have been due to the popularity that had been cumulatively built upduring the past one year by other advertising agencies who had handled its concertsin other cities and towns. If so, giving the credit solely to Ad Lib for the group’snatinwide popularity would not be justified.

Another relevant factor is the cost of publicity this year. If Ad Lib’s publicitybudget was double those of earlier concerts, the additional expenditure would not bejustified for the mere reason that the tickets were sold out within a limited period.

Even assuming that Ad Lib had played a crucial role in the success of DoubleRice’s concert in Megalopolis and it was also cost effective, how does the managerassume that Ad Lib, which might be an agency whose area of operation has so far beenlimited to this city, has the infrastructure, manpower and other resources to repeatits performance throughout the nation?

All these points need clarification before the manager’s recommendation is actedupon.

20. Balmer Island’s hired mopedsThe writer of the letter himself implies that the total number of mopeds and

bicycles being rented out now in Balmer Island by the six rental companies is limitedto 300 per day. He has not given figures of the populations of ‘owner-driven’ mopedsand other types of vehicles running in the island.

If, for instance, there are over 2,000 owner-driven mopeds and 5000 cars runningin the island, will the reduction of rented mopeds from 300 to 180 make a bigdifference to the accident statistics?

Secondly, the writer has not given figures of the total number of accidents inBalmer Island involving mopeds, and how many of these are attributable to hiredmopeds. If a predominant percentage of such accidents involve vehicles owned by theriders, there would obviously be no logical basis for the writer’s recommendation toreduce the number of hired mopeds.

Even if it is true that the hirers of mopeds have been more prone to gettinginvolved in accidents, the more effective solution is for the town council tointroduce greater restrictions regarding the persons to whom the companies can rentout mopeds to, in terms of their age, how long they have possessed a driving licenseetc., and not by reducing the number of hired vehicles.

Moreover, the reduction of hired mopeds in the island will force the tourists toengage taxis to visit the popular tourist spots, and the population of taxis willhave to be increased thereby. Since four-wheeler vehicles occupy more road space thantwo-wheelers, this will add to the traffic problems in the island, and may evencontribute to more number of accidents.

Moreover, the pollution caused by four wheelers being more than that caused bytwo-wheelers, the attraction of the island as a tourist destination may also sufferin the long run.

In support of his recommendation, the writer cites the experience of theneighboring island Torseau where there was a 50% reduction in such accidents lastyear after its town council had imposed a similar restriction. In advancing thisargument, the writer is adopting the common fallacy of holding that if Event Bfollows Event A, then Event B must have been caused by Event A, without consideringthat there might have been other more valid reasons for the occurrence of Event B.

In this case, for example, in addition to the restriction pointed out by thewriter, Torseau might have taken many other steps such as widening the roads,introducing more pedestrian crossings, introducing one-way restrictions along busyroads, increasing police surveillance etc. So, even assuming that there is some meritin the writer’s recommendation, his assertion that ‘the town council is sure toattain the 50 percent reduction’ will not come true unless Balmer also takes theseadditional steps.

American Education Aids - GRE Arguments - Model Essays 21

For the above reasons, the recommendation of the writer of the letter cannotstraightaway be acted upon by the civic authorities of Balmer Island.

21. Sartorian Company’s alpaca overcoatsThe argument advanced by the vice-president of Sartoria Company for resuming the

production of deluxe alpaca overcoats is that a new fabric supplier has been found.He himself gives the reason why the company discontinued the production of such

overcoats five years back: it had difficulties in obtaining reliable supplies of highquality wool fabric.

The vice-president does not give any specific information about the ‘new fabricsupplier’ to convince the management that the latter can be relied upon to supplyhigh quality wool fabric consistently. If the former supplier of fabric for alpacaovercoats had failed in his commitments for reasons beyond his control - such aswidespread deaths of alpaca sheep due to an epidemic, or restrictions on the importof the wool into the country - the new supplier may also fail if similar reasonsarise again.

This is the first major flaw in the note.Secondly, in order to buttress his argument that there will be a ‘pent-up demand’

for the product, the vice president says that the company has not offered alpacaovercoats for five years, and its major competitor has also discontinued theproduction of this item.

I find a couple of problems here. The first sentence talks of ‘deluxe alpacaovercoats’, while the third sentence talks of just ‘alpaca overcoats’. It is notclear whether these two categories are different or are the same. If they aredifferent, the note contains no information on why the company discontinued theproduction of even ordinary alpaca overcoats five years back. The reason could wellbe that the market for alpaca overcoats had ceased to exist because alternativeartificial fibres, which are equally elegant and are much cheaper, have emerged. Thiscould possibly explain why the company’s major competitor also discontinuedproduction of this item.

Assuming that these items are different, and there was a continuing demand forthe deluxe alpaca overcoats, the vice-president does not envisage the possibilitythat such demand has been met either by imports or by smaller manufacturers whom hiscompany had not considered as serious competitors earlier. If so, there would be no‘pent-up demand’ for the item as assumed by him.

In the last sentence, the vice-president says that the company’s profits willincrease because it can price the alpaca overcoats at a high level, in line with theprices of most types of clothing which have increased substantially over the lastfive years. The most probable reason for the increase in the prices of other types ofclothing is the increase in the price of raw materials. If the price of wool thatwill be bought from the new supplier has also gone up substantially during thisperiod, the increase in profits projected by the vice-president will not materialize.

Again, since alpaca overcoats are a luxury, it is a moot point whether theconsumers will have the necessary purchasing power for buying them after they havehad to pay escalated prices for their other day-to-day wear.

The management of the company should seek further clarifications from the vice-president on the above issues before it accepts his recommendation.

22. Steel City’s new homesThe note gives historical information relating to the past five years (namely,

that the population of Steel City had increased by more than 20 percent), and makes arecommendation that the company should build a larger number of houses next year thanit did last year.

The first flaw in this recommendation is that it overlooks the possibility thatthe needs of the additional population for housing have already been met by thiscompany and its competitors in the last few years. If so, the proposal that thecompany should build more new houses next year by employing additional workers willbe valid only if it is assumed that the population of Steel City will continue to

American Education Aids - GRE Arguments - Model Essays 22

increase. The note does not give any justification for this assumption.Secondly, the note does not say that the family incomes in Steel City are higher

than the average national family income. It only says that the ‘rate of increase’ inthe family income in Steel City is more than the national average. If the base figureof the average family income in Steel City five years back was considerably less thanthe average national figure, it is quite likely that the average family income inSteel City today is still less than the average national family income. If so, thespeculation that there will be greater demand in Steel City for higher priced housesthan for lower priced ones is not justified. This will be even more so if theincrease in the population consisted more of lower paid factory workers than higherpaid executives or managerial staff.

Thirdly, if the higher rate of increase in average household income in Steel Cityis attributable mainly to those who already had high incomes, such persons are likelyto be owners of houses already, and may not be prospective purchasers of thiscompany’s new houses.

Finally, the note also says that, by building and selling higher-priced houses,the ‘profits’ of the company will increase. However, the note cites no data to provethat higher-priced houses yield greater profits than lower-priced ones. If many ofthe higher priced houses remain unsold, the cost of carrying such unsold stock would,in fact, eat into the company’s present level of profits too.

The note itself points out that, in order to achieve the purpose advocated by it,additional workers will have to be hired. Will the required additional workers beavailable in the city? If they are, hiring them will also lead to additional cost,and may eat into the profits of the company.

For the recommendations in the note to be acted upon, the above concerns mustfirst be addressed satisfactorily.

23. Bay City’s new seafood restaurantOne of the reasons cited in the note for the recommendation to start a new

restaurant in Bay City specializing in seafood is that the consumption of seafood inthe city has increased by 30 percent over the past five years.

The note, however, does not give the base figure over which this increase tookplace.

Firstly, if seafood billing had formed only 10% of the total price of food eatenby restaurant customers five years back, it would have increased to just about 13%now. This, obviously, would not justify the opening of a special sea food restaurantin Bay City. This is also probably the reason why none of the existing restaurantowners has ventured to start a special seafood restaurant here.

The second justification for the proposal is that a nationwide study has shownthat two-income families eat significantly fewer home-cooked meals than they did adecade ago. This statement is also quite vague. If the number of home-cooked mealseaten by such families a decade ago was 55 out of 60, and this has now come down to45 out of 60, then a new specialized seafood restaurant may not be quite viable. Ifthe corresponding figures are 55 and 25, then there would be some justification forthe proposal. This point needs clarification.

The third concern that has to be addressed is whether the behavior pattern of thetwo-income families in Bay City conforms to the national pattern. I would recommend aquick local survey to validate the presumption that it is indeed so.

The note also implies that, in the opinion of two-income families, ‘eatinghealthily’ would mean ‘eating seafood’. It does not give justification for thisunwarranted presumption.

One more question that needs answer is whether a significant proportion of thepopulation would like to have exclusive seafood so often as to make a specializedseafood restaurant to be viable.

Finally, the note predicts that a specialized seafood restaurant in Bay City willprove to be both ‘popular’ and ‘profitable’. However, these two do not always gotogether. If the restaurant prices its menu with emphasis on profitability, it maylose its popularity. If it errs on the side of popularity by keeping its prices low,

American Education Aids - GRE Arguments - Model Essays 23

it may not be all that profitable.So, a much deeper study on the issues pointed out above is necessary before the

management takes a decision in agreement with the above recommendation.

24. Viva Tech’s centralized plantThe first statement of the president, that the closure of some of the small

existing assembly plants and building a large central plant will reduce costs, hasnot been substantiated with relevant facts and figures.

First of all, it is quite likely that each of the existing plants has specializedin the production of a particular equipment, has staff who are experienced in thatline, and their productivity is quite high. It is also likely that the capital costsof the existing small units have already been fully amortized. Each of these units isprobably in close proximity to the market for the particular item that it produces.If so, centralizing their production in a new plant employing new workers is notlikely to lead to a reduction in cost. The president will have to clarify thesepoints first.

The choice of Grandview for the proposed new plant is sought to be justified witha number of reasons, but these also do not appear to be sound prima facie.

The fact that Grandview has the largest adult population need not straightawayimply that the proposed factory will get employees easily and quickly. If a largeproportion of this adult population are aged above 50, and those of lesser age arealready employed, this prognosis will fail. For validating the presumption in thenote, the percentage of unemployed population in the age group preferred by thecompany will have to be ascertained first.

It is stated that the company produces ‘high tech’ medical equipment. Itobviously needs highly skilled workers in its employment. How many among theunemployed population in Grandview will fall in this category is also a question forconcern.

Thirdly, the wage payable to such highly skilled workers, if at all they areavailable locally, cannot obviously be determined on the basis of the present averagewage level of workers in Grandview, because most of these workers may be employed inlow tech industries.

Fourthly, the note refers to the moratorium in city taxes for three years offeredby the Grand View’s town council as another point in favor of locating the new centerthere. Again, the note does not quantify the consequent benefit to enable us to judgewhether this should be a major consideration in deciding the location of the plant.If the city taxes form a minuscule 0.1% of the total annual expenses of the proposedfactory, this concession should not play any role in decision-making. If it forms 40%of the total annual expenses, there would be justification for the recommendation.The president must clarify this point too.

The price of land should be a major consideration in deciding upon the locationof the proposed factory. The note does not indicate the price of land in Grand Viewvis-a-vis the price in other competitive locations. It does not also indicatewhether the other town councils, where it is possible to get cheaper land, were alsoapproached for concession in taxes similar to what is offered by Grand View towncouncil.

The note of the president is deficient in a number of respects as pointed outabove, and he should be called upon to examine the matter de novo and present a freshnote.

25. Hopewell’s new golf courseThe major flaw in the Hopewell mayor’s memo is the classical one of presuming

that if event B follows event A, then event B must have been caused by event A.He attributes the increase in tourism, increase in the number of businesses and

the increase in the tax revenues of Ocean View in the last two years solely to thebuilding of a new golf course and a new resort hotel there.

While it may be logical to presume that the building of a new golf course and anew resort hotel led to an increase in tourism and also some increase in tax

American Education Aids - GRE Arguments - Model Essays 24

revenues, the increase in the number of new businesses and a 30% increase in taxrevenues could well have been due to other reasons such as an increase in the rate oftax, or the imposition of a new type of tax. The note does not even try to explorethese reasons.

In fact, even for the observed increase in tourism, there could be other reasonsnot explored by the mayor, such as the starting of a number of new industries, theconstruction of a large number of houses etc.

The mayor also does not assert that the terrain and climate of Hopewell wouldadmit of the construction of a golf course there. As its name suggests, Ocean View isprobably a beach resort and has been attracting tourists even earlier, and the newgolf course is only an additional attraction for them. It is not likely that a meregolf course attracts tourists to a place.

While stating that the tax revenues of Ocean View had increased by 30% in thelast two years, the mayor does not give comparative figures of the tax revenues ofthe two towns. If the tax revenue of Ocean View two years back was $50,000, it wouldhave since increased to $65,000. If Hopewell’s tax revenue even now is $1,000,000,then an increase of $15,000 that could be generated by a new golf course and a newresort hotel would not be of much consequence to this town.

Note that the mayor does not say that Hopewell should build a better golf courseand a better resort hotel than Ocean View, but suggests the construction of ‘similar’ones. If Ocean View has already enrolled a majority of the prospective clients forthese facilities, why should they now abandon Ocean View and come to Hopewell?

Finally, the note would have logical validity if it had said that a new golfcourse and a new resort hotel would result in some increase in tourism in Hopewellalso. But the mayor recommends these measures as the ‘best’ way ‘to improveHopewell’s economy’ and ‘generate additional tax revenues’. To validate hisstatement, he should have discussed other alternative proposals and shown how hisrecommendation is superior to all of them for achieving these results.

As it stands now, the recommendation of the mayor has serious lacunae and cannotbe acted upon straightaway.

26. Saluda’s music directorThe memo from the chairperson of the School Board seeks to persuade the

management that Charles Schade, present music director at Steel City High School,would be the most suitable person to plan and direct the general music educationsystem of the entire Saluda music system.

But a few concerns should be clarified by her before her recommendation isaccepted.

The first justification cited by her is that, during the past 5 years that Schadewas music director there, the Steel City High School won three regional bandcompetitions. The presumption that this feat of the school band was solely due to theefforts of Schade is not substantiated. It is quite likely that this school alreadyhad a strong music tradition established by its former music teachers and Schade hadmerely followed their footsteps. In fact, if the school had won the band contestduring the first three years after he took over and had lost it during theimmediately preceding two years, it would even mean that the earlier winning streakof the school had been broken because of his incompetence. If this be the case, hecould hardly be considered suitable for a more challenging assignment relating to awhole district.

Similarly, the improvement in the music rehearsal facilities and musicalinstruments in Steel City High School during the last five years might have been dueto the initiative taken by someone other than Mr. Schade, such as the Principal orthe President of the School Committee, and Mr. Schade might have merely implementedit. If this be the case, the full credit for them being attributed to Mr. Schade maynot be justified.

Even assuming that Mr. Schade has been an outstanding music director of SteelCity High School, his success may have been due to his being a good music conductoror band master himself, or a personal motivator of individual musically-talented

American Education Aids - GRE Arguments - Model Essays 25

students. Will such talents be sufficient or even relevant for manning a post whosemain function is planning and directing the general music education programs for anentire school system of a city?

Improvement of music education in a single school would have required arelatively small financial outlay, but the total expenditure for an entire schoolsystem is likely to be substantial. Does the Saluda school system have enoughbudgetary provision to implement a music education improvement in the entire schooldistrict within a short period?

Finally, the note does not discuss any alternative name for the post. Is therenoone within the Saluda school system or in any other neighboring school whosecredentials are even superior to those of Mr. Schade and who can also be consideredfor this position?

Finally, has Mr. Schade been sounded at least informally and his consent obtainedfor the proposal?

These questions will have to be satisfactorily answered before the recommendationin the memo is acted upon.

27. Automate Inc’s plan to retain its employeesThe note of the president of Automate may be understood in two ways: (i) It is

only by offering salaries equal to those paid by Sparks that Automate can retain itsbest employees; (ii) By offering salaries equal to those paid by Sparks, Automate candefinitely retain its best employees.

The first point to be noted is that Automate is an automobiles manufacturer whileSparks is a general manufacturing company. While some operations in both thesecompanies may require identical skills, not every worker in Automate may qualify fora job in Sparks. It is quite likely that those who have already left Automate to joinSparks possessed these common skills and preferred a company which paid them highersalaries. If the others are not likely to be offered jobs by Sparks either in theirfirst plant or subsequent plants for the reason that their skills do not matchSparks’ requirements, there is no point in Automate increasing its salary bill. Thisissue must be clarified first.

Secondly, it is not the quantum of salary that alone motivates an employee tocontinue to work in a company. The location of the plant is a major factor. IfAutomate is located in a major town with a good climate and all infrastructuralfacilities such as good schools, good hospitals, entertainment avenues etc, and thenew plants of Sparks are going to be in greenfield sites without these attributes andin less congenial climates, the best employees of Automate may not be lured merely bythe higher salaries offered by Sparks.

Moreover, it is likely that the spouses of most of the employees of Automate arealso employed in the same town and will lose their income if the family relocates inSparks’ place. In this case also, a higher salary for one of the spouses in Sparksmay not be a great attraction for the family to relocate at the new place.

On the other hand, if most of the workers of Automate do possess skills which aresought by Sparks also, and the locations of Sparks’ factories are also equallyattractive, then merely matching of salaries may not guarantee that they continue tobe loyal to Automate. This company will have to match all other non-quantifiablefacilities that Sparks offers to its employees such as a better environment, morecongenial work culture and more up-to-date equipment.

The above points must first be clarified before the Automate Board of Directorstake a decision to irrevocably increase their salary bill.

28. Mylar’s plan for free breakfast for studentsBased on a study involving 100 children in the age group 5 to 12 who were

provided breakfast at school, the superintendent of Mylar school district advocatesthe provision of breakfasts for all students before each school day.

He claims that this would reduce absenteeism and tardiness, and improve academicperformance in all of Mylar’s elementary and secondary schools.

The justification cited by him for the suggestion is that the study showed that

American Education Aids - GRE Arguments - Model Essays 26

children on the breakfast plan were more regular in attending school and were moreprompt in arriving on time.

The first flaw in his suggestion is his presumption that the experience with 100students can be extrapolated to the entire student community in the district. If thetotal number of students in Mylar district is 5,000, a mere 100 among them would betoo small a sample for influencing a decision involving major financial implications.

The second problem is that he presumes that these 100 students wererepresentative, in terms of their economic conditions, of the entire studentpopulation of Mylar district. If these 100 students were from the poorest strata ofthe Mylar society and could not afford healthy breakfast at home, the experience withthem cannot be extrapolated to all other students most of whom may be affluent enoughto have much better quality of breakfast at home and may not care for the breakfastsupplied in school.

We may readily agree with the superintendent’s statement that children whoregularly eat a healthful breakfast tend to perform better in school. But hisrecommendation that, therefore, breakfast must be provided in schools throughoutMylar district ‘to improve academic performance’ assumes that most of these studentsdo not take breakfast at home now. This is obviously an unreasonable presumption. Ifmost students are already used to having a healthful breakfast at home before theycome to school, then replacing home breakfast with school breakfast will hardlyimprove their academic performance.

The age group of the 100 children involved in the study was 5 to 12, while moststudents of secondary school would be in the age group 13 to 16. One other flaw inthe superintendent’s recommendation is the extrapolation of the behavior of the firstgroup of students to the second group also.

The superintendent does not indicate the source from which the additionalexpenditure involved in his suggestion will be met. Will the local body be preparedto meet the cost, or will the parents be asked to pay for the breakfast to beprovided by the school? If the latter be the case, has he ascertained the willingnessof at least a majority of the parents to do so?

The superintendent must be asked to give satisfactory answers to all thequestions in an additional memo before the authorities can agree to his suggestion.

29. Ida McAllister as next MayorIn recommending that Ida McAllister should lay emphasis on environmental causes

for winning the next Mayoral election, the author of the note presumes that thevoting preference of Lake City citizens is predominantly based on the candidates’stand on environment. There is no information in the note for supporting thispresumption.

Assuming that environment is a major concern of the voter, and Ida’s defeat lasttime was only because her successful rival was seen as being more committed toenvironmental causes than she was, it is quite likely that he has been aggressivelyimplementing environment-preservation schemes during his tenure. If so, why shouldthe voter now decide to vote him out and install in his place a new convert to thecause?

In support of his recommendation that, in order to prove her commitment toenvironmental cause, Ida should promise to close all Lake City’s parks to automobiletraffic, the author of the note cites the example of Alecia Yu, a former mayor ofanother city Plainsville, who had taken a similar step there on the advice of a‘small group of concerned citizens’. This example raises the doubt that, though thisstep might have had the approval of this small minority who credited him withimproving the quality of life in Plainsville, it was really unpopular with a muchlarger number of voters who were earlier conveniently driving through these parks toreach their places of work and suffered gross inconvenience because of the ban. Infact, this might have been the reason why Yu could not win the re-election and endedup as a ‘former’ mayor.

If this be the case, an aspiring mayoral candidate announcing such a potentially

American Education Aids - GRE Arguments - Model Essays 27

unpopular step even during the election campaign would be suicidal for her, and mayresult in her securing less than even the 30% vote she got last time!

Again, on the basis of the fact that most citizens of Lake City participate inthe community’s recycling program, the writer presumes that they would all support anannouncement to close all Lake City’s parks to automobile traffic. Taking part in therecycling program does not affect anyone adversely, whereas closing the parks toautomobile traffic may cause considerable inconvenience to a large number ofcommuting citizens. So, the support for the former does not ipso facto imply an equalsupport for the latter.

Finally, the assertion that the announcement would ‘guarantee McAllister’ssuccess in her current quest to be Lake City’s new mayor’ does not take intoconsideration the possibility that the citizens of Lake View would be more concernedwith the candidates’ stand on other problems of the city such as water supply,economic development, education etc.

For the foregoing reasons, Ida McAllister would do well not to go by therecommendation in the note during her election campaign, and steer clear of any talkabout banning driving through the parks .

30. Elliottown’s tourism recordThe purpose of the note is to plead for a significant increase in the funding for

the Central Park and Museum of Modern Art that opened last year in Elliottown.But the justifications given by the author for this recommendation are hardly

convincing.Firstly, he himself quotes the local gazette to show that the number of people

who travel to the town has increased significantly ‘over the past several years’,namely even years before the park and the museum were opened. Therefore there must bea strong reason other than last year’s opening of the park and the museum that hasbeen the cause for the increasing numbers of travelers to the town even last year.

The figures quoted by him are that, during the last three years, the numbers ofpeople who ‘arrived at the local airport’ were 40000, 80000 and 100000, while thenumbers ‘received at the local train station’ were 20000, 40000 and 50000.

Note that the author does not corroborate these figures of arrivals at transitpoints in the town with evidence of a corresponding increase in the number of hotelrooms and their occupancy ratios. This gives rise to the suspicion that theseadditional Travellers are merely passing through Eliottown, and are headed towards apopular tourist center which can be reached only via this town, and which hasimproved on its attractions in the last three years. If this is the real state ofaffairs, there is hardly any justification for the claim that ‘tourism’ in Eliottownhas increased, and the consequent attribution of such false claim to the opening of acentral park and a museum there.

Commonsense also dictates that a park is mainly attractive only to localresidents and hardly ever enthuses hordes of outsiders to swarm to it, unless it isknown for its ancient or rare trees such as the Redwood Forests in California with2500-year-old Sequoia trees. How a mere one-year-old park in a small town can attracttourists to visit it is beyond one’s comprehension.

The author also tries to bamboozle the reader with annual figures of travelersthat run into thousands. But break these figures down to their daily average, andwhat do you get? The increase in daily arrivals in airports is from 110 to 220 to280, and that at the railway station is from 55 to 110 to 140! These figures canhardly be said to put Eliottown on the tourist map of the nation!

It is even likely that quite a substantial number of the increase in arrivals inboth the airport and train station in Eliottown are accounted for by travels ofEliottown’s own residents.

So, while Eliottown’s park and museum may deserve additional funding, it is notfor the reasons enumerated by the author of this note.

American Education Aids - GRE Arguments - Model Essays 28

31. Cell phone use by driversThe argument of the author is that only some drivers using hand-held cell phones,

and not a majority of them, cause problems on the road including serious accidents,and that therefore it is sheer folly to legally prohibit the use of such cell phonesby drivers.

The perversity of this argument is obvious because it can be applied equally todrunken driving, or driving by minors, or driving without a license. Not everyoneindulging in these activities causes a serious accident. Would the author of thisarticle argue equally vehemently that drunken driving or driving without licenseshould be legalized?

The author of this editoriaal has clearly misunderstood the rationale behindprohibiting the use of handheld cell phones by drivers. It is true that the phonecall distracts the driver, but that is not the reason behind the ban. The real reasonis that, while using the handheld cell phone, the driver has the use of only one handfor steering the vehicle. Driving requires the continuous use of both hands on thesteering wheel, in order to have control over the vehicle in the case of anunforeseen an emergency. The parallels cited by him, of listening to a radio ordisciplining children, though they could be distractive, do not lessen the physicalcontrol of the driver over his vehicle. So, equating the use of handheld cell phoneby a vehicle driver with listening to radio has no logical force.

Moreover, a driver has to have his eyes always on the road. That is why fixing atelevision set in the front panel of a vehicle has been banned, even though there isno such prohibition for a radio. The ban on the use of handheld cell phones bydrivers has a similar rationale.

The last line in the article contains an equally spurious logic. Noone isproposing to ban the use of cell phone which is indeed a convenient device,particularly during emergencies. If the driver is faced with such an emergency, hecan certainly stop the car, pull it to a side, and use the cell phone to seek help.

The recommendation in the article is thus based on false assumptions andillogical parallels, and deserves no consideration.

32. Hiparia’s shopping mallsEven the fact cited by the author is that people in Hiparia are just ‘beginning’

to do most of their shopping by ordering merchandise from mail-order catalogs and theInternet. It is quite likely that this could prove to be a beginner’s fad; people mayagain resort to shopping in malls because they might feel that they are missing thepleasure of looking at and touching a large variety of items before they decide tomake a purchase. If this happens, his prognosis that there will be a reduction in theconsumption of vehicle fuel in Hiparia will not materialize.

Secondly, even assuming that people will continue to shop by ordering items frommail-order catalogs and the Internet, reaching these items to them will involve theconsumption of fuel by the delivery services which generally use bigger vehicles thanthose of consumers. The author does not cite relative statistics of these two typesof consumption of fuel to enable us to decide whether there will be a net increase ora net reduction in such consumption.

Again, using mail order or Internet for purchase of merchandise is possible onlyfor planned items. People will still have to make across-the-counter purchases whenthey require fresh items such as milk or bread, or emergency medicines etc. Suchpurchases would continue to require travel.

The author also assumes that people visit malls only for buying articles.Shopping malls usually house a number of other services such as restaurants, beautyparlors and gymnasia, and people generally combine the use of these services alongwith shopping. Even if they do their merchandise shopping through mail order orInternet, they will continue to visit the malls for these purposes. Consequentlythere may not be any appreciable reduction of consumption of fuel by them.

Finally, the author assumes that the people will not use the time saved in nothaving to visit shopping malls by undertaking other travels which may require evenhigher fuel consumption. The validity of this assumption is also open to question.

American Education Aids - GRE Arguments - Model Essays 29

For the above reasons, the forecast of the writer that there will be a netreduction in the consumption of vehicle fuel in Hiparia is not likely to materialize.

33. Ceramic pots at prehistoric sitesThe dispute among archaeologists, according to the author, is on whether the

distinctively shaped ceramic pots scattered in prehistoric sites over a wide areawere spread through the migration of potters or through trade.

Having referred to this dispute, the author makes a categorical assertion that ananalysis of prehistoric human skeletons can settle this debate.

The first flaw in his argument is that he cites no authority for the assertionthat bone-analysis is the final arbiter to decide whether the person concerned was alocal resident or a migrant.

Secondly, even assuming that he is correct on this count, he does not cite anyauthority for his assertion that a single parameter, namely the presence or absenceof a particular metallic content in the bones, can settle the issue once for all.

Even assuming it is so, his further presumption that the bones found near thepots at a few sites were those of the potters and not of the users of the pots iswithout any rational basis. If the latter is true, the pots themselves could wellhave reached these places through trade, and were only used by the local residents,and not made by them.

Another unwarranted assumption that he makes is that the metallic element in thebones had only come from the food eaten by the persons concerned. He has notconsidered the possibility that it might have been due to genetic reasons or fromsome drug that they had taken during a particular epidemic or some other similarreason.

Again, his assertion that high levels of the metallic element contained invarious foods are strongly associated with ‘people who migrated to a new place afterchildhood’ is itself illogical. While a particular group of migrants may have beenfond of a particular food, how can anyone believe that all persons who decided tomigrate from one place to another had developed a liking for that food?

Even if the author’s theory is accepted, the metallic content would be found inthe bones of only the first generation of the migrant potters. Does he want us tobelieve that all the bones found near the pots were those of such first generation ofmigrants?

Even according to the author, the bones with high levels of the metallic elementwere found near the pots at only ‘a few’ prehistoric sites. The extrapolation of thefinding relating to a few sites to ‘various prehistoric sites scattered over a widearea’ is again irrational.

The basic raw material for the potter’s trade is a suitable type of clay. If thepotters had, in fact, migrated to various historic sites scattered over a wide area,it must first be shown that the type of clay required for the distinctively madeceramic pots was available at all these sites. The report is silent on this point.

Families migrating from their usual place of residence to a new place can beexpected to carry their possessions required for personal use in the new place. Theydo not usually carry their stock-in-trade during such migrations, particularly whenthe stocks are such fragile items as mud pots. Note that the report relates to‘prehistoric sites’: were the means of transport during those days conducive to thesafe transport of mud pots by migrants?

The thory of the author bristles with many infirmities as above, and cannot beaccepted.

34. Osteoporosis and milk productsThe second sentence says that osteoporosis is a disease that is linked to both

environmental and genetic factors, and that intake of milk and dairy products whichare rich in vitamin D and calcium ‘can’ help prevent osteoporosis in old age.

The next sentence talks of the result of a study of ‘a large number’ of peoplewhich showed that those who had consistently consumed dairy products throughout theyears of study had a higher rate of bone fractures than any other participants in the

American Education Aids - GRE Arguments - Model Essays 30

city.One question that arises immediately is whether the ‘large number’ (which has not

been specifically quantified) was really large enough and representative enough tolead to a general conclusion about the entire population as adumbrated in the largesentence. If the ‘large number’ was just a few hundreds, then the effect of dairyproducts on them cannot be extrapolated to the whole of humanity.

Secondly, if those among the sample population who had been taking dairy productswere also those whose genetic background and environment were more conducive to theirdeveloping osteoporosis, then the higher incidence of bone fracture among them may bedue to these factors and not to their consumption of dairy products. It could even beargued that, if they had not been consuming dairy products, their susceptibility tobone fractures might have been even greater than what was observed.

Similarly, if the persons in the sample group who exhibited bone fractures weremuch older than those who did not, even then the bone fractures might have been dueto their age and not due to the consumption of dairy products.

Again, the author refers to only the ‘rate of bone fractures’ and does notmention the actual numbers of ‘those who consistently consumed dairy products andalso had bone fractures’ and ‘the other participants’. It is quite likely that theformer group consisted of only 100 persons and 52 of them (or 52%) had suffered bonefractures, while the latter group (who did not consume milk or dairy products)consisted of 5000 persons and 2500 of them (or 50%) had suffered bone fractures. Ifso, the author’s conclusion that consumption of dairy products increases the risk ofosteoporosis will be totally unsound.

Moreover, the author himself says that bone fracture is ‘a symptom’ ofosteoporosis. This means that every bone fracture may not have been caused byosteoporosis, and could have been due to other reasons such as cancer of the bone. Itis quite likely that the bone fractures of a majority of the persons observed in thestudy might have been due to such other reasons and not osteoporosis.

For the foregoing reasons, I would not be deterred by what this author says, butwill continue to consume milk and dairy products!

35. Mentia residents and salicylatesThe fact that salicylates are members of the same chemical family as aspirin does

not immediately imply that salicycates can be used as a drug substitute for aspirin.Had this been so, drug companies would long ago have started producing pills based onsalicycates, particularly when this is ‘richly available’ in many foods. The factthat it is only food processing companies and not drug companies which use them wouldimply that salicycates, while they are good food preservatives, do not lendthemselves to conversion as drugs.

As the author of the note says, aspirin itself is not used as a ‘preventive’ forheadaches, but is used only to ‘treat’ them. He claims that the rise in the use ofsalicycates by the food processing industry has been contemporaneous with ‘lesserincidence’ of headaches in the sample population. According to the writer’s thinking,salicycates are even more potent than aspirin because they are even able to ‘prevent’headaches! If so, the drug companies must have by this time directly exploited thispreventive potential of salicycates rather than leaving it to food processing firmsto use it as food preservatives or flavour additives. Since drug companies have notdone so, their own experiments must have contradicted the conclusion in this note.

For arriving at his conclusion, the author correlates the ‘rise in commercial useof salicylates as preservatives by food-processing companies’ to ‘a steady decline inthe average number of headaches reported by participants in our 20-year study’. Notethat he does not even assert that the participants in the study had been regularlyconsuming the food items containing salicylates. If most of the participants in thestudy had not been consuming such food at all, the bottom is knocked out of theauthor’s conclusion.

Since many food items are ‘naturally rich’ in salicycates, the author must alsoclarify whether the amounts added as preservatives to the food items are of suchlarge magnitude as to make a difference to the propensity of their consumers to

American Education Aids - GRE Arguments - Model Essays 31

suffer headaches.Finally, in forecasting that the use of salicycates as flavor additives in food

items will lead to a reduction in the incidence of headaches suffered by the averagecitizen of Mentia, the author presumes that these food items will become popular withthe average citizen of Mentia. He does not give any supporting evidence to justifythis presumption. If a large percentage of Mentia’s population are used to consumingfreshly cooked food and not preserved food, the author’s prognosis will not cometrue.

For the foregoing reasons, I am not in a position to agree with the conclusion inthe given note.

36. Dr. Karp on Dr. FieldOn the basis of his ‘recent’ interviews with children in the ‘group of islands

which includes Tertia’ (the results of which are contradictory to the conclusion ofDr. Field that had been based on his observations in the island of Tertia twentyyears ago), Dr. Karp comes to two conclusions: (i) Dr. Field’s thesis about Tertianvillage culture is false, and (ii) the observation-centered approach to studyingcultures is invalid.

Dr. Karp must first tell us what percent of the number of children he hadinterviewed in the island group were from Tertia. If a predominant percent of theinterviewees had belonged to other islands in the group, their experience with theirbiological parents cannot be taken as applicable to Tertian children also. This isbecause the cultures of all tribes in different islands even in the same geographicalregion need not be identical.

Secondly, the fact that children talked more about their biological parents thanabout other adults in the society does not by itself nullify Dr. Field’s conclusion.It is quite possible that, even while the entire village takes responsibility forrearing all the children there, the children themselves spend relatively more timewith their respective biological parents. If this be the case, it is not surprisingthat they are prone to talk more about the parents than about others.

Thirdly, Dr. Karp criticizes a 20-year-old finding of Dr. Field on the basis ofhis recent study. It is quite likely that both are correct - the child rearingpattern in Tertia might have changed significantly in the last twenty years forvarious reasons. At best, this is the logical conclusion that Dr. Karp can come to.His assertion that ‘Dr. Field’s conclusion about Tertian village culture is false’ isnot only illogical but is even unethical. No wonder he has been named ‘Dr. Karp’!

Even if we agree with Dr. Karp that Dr. Field had indeed been wrong in hisconclusion about child-rearing pattern in Tertia, he is not justified in making thesweeping statement that the observation-centered approach to studying cultures isinvalid. He himself concedes that Dr. Field is a ‘noted anthropologist’. If Dr. Fieldhad been using the observation-centered method for coming to his conclusions and ifthis method was so blatantly wrong, he could not have gained reputation as a ‘notedanthropologist’!

Since the author has not conclusively established that the ‘interview-centered’method is superior to the ‘observation’ method, the assertion in his last sentencealso does not have force. His assumption that the result of a study in Tertia can beextrapolated to all other island cultures is even more invalid.

37. Palea village and its basketsOn the basis of the discovery of a single ‘Palean’ basket in Lithos, the author

comes to the conclusion that such baskets were not unique to Palean people as isgenerally believed.

The first presumption of the writer is that because Palean people were notcapable of taking such baskets to Lithos, the basket found in Lithos must have beenmade locally. He does not cite any basis for this presumption. It is quite likelythat the Lithos people were more enterprising and had the knowledge of making boatsby which they had crossed the Brim river and taken the Palean baskets, along withother commodities, back to their own place. If this could account for the discovery

American Education Aids - GRE Arguments - Model Essays 32

of the single basket in Lithos, this item could still have been unique to Palea.The second assumption of the writer is that Brim River had never got dry, but he

does not cite evidence for this presumption. If this river, like most rivers all overthe world, was flooded mainly during rains but could become dry or shallow duringother seasons, Palean people could have easily crossed it for trade and for otherpurposes even without having to use a boat. This could also account for the discoveryof a Palean basket in Lithos, and could still be consistent with the uniqueness ofthe basket to Palea.

Moreover, lack of archaeological evidence available now is not conclusive proofthat Palean people did not know of the use of boats. It is not necessary to haveboats ‘capable of carrying a group of people and cargo’ to transport a mere basketacross a river. Palean people probably were using relatively smaller boats for theirtrade, and these had been made of such perishable items that no archaeologicalevidence about them is available now.

Finally, the argument that Paleans would have had no need to cross the riverbecause the woods around Palea are full of nuts, berries, and small game is obviouslyunsound. Even ancient people did not live on ‘nuts, berries and small game’ alone.They would certainly have needed cereals as well as clothing materials. If these wereavailable only across the river, they would certainly have discovered the means ofcrossing it, and could have bartered their unique baskets for purchasing these items.

Even on this point, the author says that the woods around Palea ‘are’ full ofnuts, berries and small game. On what evidence does he presume that the conditionsaround Palea had remained the same during prehistoric periods?

For the foregoing reasons, I would not agree with the conclusion of the author ofthis note.

38. West Meria and fish foodThe major flaw in the memo is that it seeks to arrive at its conclusion from a

comparison between the ‘number of visits to doctors in East Meria for treatment ofcolds’ with the ‘number of excuses given in West Meria for absences from school andwork’. If most of the latter are false and are spuriously given only for the purposeof malingering, the very assumption that people of East Meria are less prone to coldsthan those of West Meria would be wrong. If medical records are scrutinized, it mayprobably be found that people of West Meria are also visiting doctors for treatmentfor colds only once or twice a year.

On the same token, if school and factory records of East Meria are scrutinized,it would most probably be found that absenteeism due to alleged colds is as rampantthere as it is in West Meria!

Even assuming that people in East Meria are less prone to attacks of cold thanthose in West Meria, in attributing this phenomenon solely to a ‘high fishconsumption’ there, the memo suffers from the classical fallacy of connecting twoobserved phenomena as a cause and its effect, without considering the possibilitythat the two may be totally unrelated, and that there could be other more validreasons for the observed result.

The more agreeable climate in East Meria could possibly the real reason for lessincidence of cold there. Also, since most colds are caused by allergy, a relativelyless polluted atmosphere in East Meria could be another real cause. The author musthave discussed and ruled out all such causes before making his assertion.

Moreover, the author does not cite figures to prove that the present averageconsumption of fish in West Meria is less than that in East Meria.

Finally, even assuming that consuming fish does reduce one’s susceptibility tocolds, whether a fish oil derivative such as Ichthaid will have the same effect asfish as a direct food is also open to question.

Even assuming that the daily consumption of Ichthaid will prevent colds in WestMeria, it does not follow that workers and students who wish to malinger will notcontinue to cite cold as the alleged reason for their absence. So, the assertion inthe memo that consumption of Ichthaid will lower absenteeism in work places andschools is not likely to materialize.

American Education Aids - GRE Arguments - Model Essays 33

For the above reasons, I am not in a position to agree with the recommendation inthe memo straightaway.

39. Omni University & Humana CollegeIn this memorandum, the president of Humana College recommends that the college

should emulate the nearby Omni University in starting long-distance degree programs.The first flaw in the memorandum is the comparison of a percentage figure

relating to just one year of Omni University with the absolute figures of HumanaCollege relating to a period of three years.

If the number of students in long-distance degree programs of Omni University hadmerely increased from 100 to 150 in one year, while the enrollment at Humana Collegehas decreased from 1000 to 960 in three years, there would really be no cause forconcern for the latter’s management, and Omni would certainly not be an ideal toemulate.

The president also has not ascertained whether, even while the enrollment inlong-distance degree programs in Omni University had increased, the enrollment in itsclassroom programs had increased or decreased. If there had been a considerabledecrease in its classroom programs last year, then again Omni is not fit to beemulated by Humana.

Even in respect of expenditures on classroom and dormitory, it is quite likelythat Omni University had incurred substantial capital expenditure in the previous twoyears and had therefore no need to incur much expenditure for this purpose last year.For a logical comparison, the expenditures incurred by both institutions on similaritems during the same three-year period must first be ascertained.

Another point that needs to be studied is whether the courses offered by OmniUniversity and Humana College are comparable. Not all courses lend themselves todistance learning. If most of the courses offered by Humana College belong to thiscategory, then the step suggested in the note will not be successful.

For classroom enrollments, a college usually competes with only a neighboringuniversity. But, in enrolling students for distance learning, a college has tocompete with all universities across the nation offering similar courses. Does theHumana College have faculty and other resources to successfully compete with allother universities in the country that are offering distance learning? Will a degreeawarded by a mere college have the same prestige or marketability as one awarded by auniversity?

These concerns must first be addressed satisfactorily before the recommendationin the note can be acted upon.

40. Excello Food Markets & organic foodFirst of all, the recommendation in the memo is based on the fact that, in 90

towns in which the national chain Excello has branches, it is facing competition fromnatural-food stores. In how many towns and cities throughout the nation does Excellohave branches and how representative are these 90 towns for influencing a nation-widedecision?

There is no gainsaying the fact that concern about pesticides and chemicalpreservation is increasing and that more and more people are gradually switching overto organic food products. Thus, any food store would do well to provide this optionto its customers.

But the assertion in this memo that this has to be done ‘to increase our profits’implies that the margin of profit in buying and selling organic food items is greaterthan in trading in conventional food items. The note does not give facts and figuresto justify this assertion. If the profit margin on organic products is much less thanon conventional items, it is quite likely that the competitive outlets openedrecently will become non-viable and close down soon. In such a scenario, the betteroption for Excello will be to continue its conventional food items and, in addition,open a special counter where organic food products are made available for catering tothe special clientele.

The fact that a large proportion (75%) of the visitors to the Excello booth in

American Education Aids - GRE Arguments - Model Essays 34

Sun City fair asked for ‘free’ samples of organic fruit implies that they were notwilling to pay for it. If this group is considered representative of the prospectivecustomers of organic food items, it is not likely that the profits on organic foodwould be more than the profits on conventional food items.

Also note that this group asked for organic fruit alone and not other preservedorganic food items. So, their behavior cannot assure high sales of organic foodproducts.

I would, however, be inclined to agree that, in order to cater to a new type offastidious clientele, Excello must open special counters where organic food items aresold, but not necessarily because of the unsubstantiated claim that these items willgenerate more profit.

41. Fly-Away Pest-Control Company’s efficiencyIn this memo, the vice-president of the food-distribution company recommends the

re-hiring of the more expensive Buzzoff Company in place of Fly-Away Pest-ControlCompany for providing pest control services in its Palm City fast-food warehouse.

The recommendation of the vice-president would have been more persuasive if hehad established that the warehouse in Palm City and the warehouse in Wintervale arestrictly comparable both in terms of the types of items and the total quantitiesstored in each.

The memo itself makes a distinction between the types of food stored in the twoplaces. The first is mentioned as a ‘fast food’ warehouse while the second ismentioned merely as storing ‘food’. If ‘fast food’ is inherently more susceptible topest damage than just ‘food’, then a comparison of the efficiencies of the twoagencies on the basis of the reported results would not be justified.

Secondly, the note does not mention the different quantities of food stored inthe two warehouses. If the former had $5 million worth of food items and suffereddamages of $20,000, while the latter had $40,000 worth of food items and suffered adamage of $10,000, it is the former agency which should be considered to be moreefficient.

Thirdly, the climate of a place and the surroundings of the warehouse also play arole in pest attack. Are the warehouses in Palm City and Wintervale comparable inrespect of these two parameters? If the former is more prone to pest attack than thelatter, even then one cannot come to the conclusion that Fly Away is less efficientthan Buzz Off.

Fourthly, the note says that Buzz Off had been used for many years by thiscompany. If so, its past record relating to the Palm City warehouse must be availablein its archives. The vice president will have to show that, other things being equal,the record of Buzz Off in Palm City warehouse in the earlier years was superior tothat of Fly Away last year. This would, in fact, be the acid test for comparing theirrelative efficiencies.

Finally, even assuming that engaging Fly Away is likely to result in more pestdamage, it should be examined whether this additional damage gets set off by thelower fee payable to it, so that there is still a net saving of money to the company.

As it stands, the recommendation of the vice president appears to be motivatedand not objective. The management should ascertain additional facts as pointed outabove before it acts on it.

42. Paraterra’s attempt to promote ecotourismThe note does not establish beyond doubt that the advertising campaign of

Bellegea to promote ecotourism had indeed been successful. The statement that thenumber of foreign visitors arriving at its main airport ‘doubled’ does not meananything, unless it is quantified. If the number had doubled from just 200 to 400,the campaign cannot be said to have been successful, whereas if the figures were20,000 and 40,000, there would be substance in the given argument. The minister mustfirst answer this question satisfactorily.

Secondly, he does not establish that the increase in arrival of foreigners inBellegea’s main airport was caused by the promotion of ecotourism. He does not

American Education Aids - GRE Arguments - Model Essays 35

convincingly rule out any other reason for such increase such as increased economicactivity, the establishment of new convention centers, the promotion of a new golfcourse or a Disneyland type of fun center etc.

The same holds good for the 10% increase in the per capita income of thatcountry. Could there not have been other more valid reasons for such increase, suchas the establishment of a new industry? The note is silent on this point also.

Assuming that the campaign had indeed been successful in Bellegea, on what basisdoes the minister argue that an identical strategy will succeed in Paraterra? He doesnot assert that the two countries are similarly placed in respect of ecotourism. Onthe contrary, he says that Paraterra is a ‘tiny country’, implying that Bellegea isnot so tiny. So, what worked in the case of the latter may not work in the case ofParaterra.

His recommendation in the last sentence that the current director of Bellegea’sNational Tourism Office should be hired as the consultant for the proposed campaignis not quite sound. It is quite likely that this functionary would consider Paraterrato be a rival to Bellegea for garnering the tourists’ dollars. In this case, therewould be a conflict of interests between his allegiance to his own country and hisobligation as a consultant to Paraterra. If this happens, one cannot expect that hewould give his best attention to the promotion of ecotourism in this rival country.

For the foregoing reasons, I would not agree with the recommendation of thefinance minister straightaway.

43. Rialto and Apex TheaterThe first assumption of the manager of Rialto Theater is that the pool of

moviegoers is declining. He justifies this assumption on the basis of the views of’85% of the respondents to a recent survey’ that the high price of newly releasedmovies prevents them from going to the movies more than five times a year.

The note does not quantify the population of the town, the number surveyed andthe number of those who responded to the survey. If the population of the town is100,000, of whom 1000 persons were surveyed and only 100 of them actually respondedto the survey, then the reported view would be that of just 65 out of 100,000, andcannot be considered to be a good enough sample to be acted upon.

Secondly, the note also does not give the economic profile of those who weresurveyed and who responded, and whether they are representative of the customarypatrons of Rialto Movie Theater.

Assuming that they are, there is an inherent contradiction in the manager’srecommendation. The capital expenditure on plush carpeting and seats as well as on anew sound system proposed by him would, in fact, necessitate further increase in theadmission price of Rialto. This would make going to movies even less affordable toits prospective clients. In fact, his recommendation to make big changes at huge costmay accelerate its ‘closing for ever’, a prospect he finds threatening.

Assuming that the theater has 200 seats with a break-even attendance of 100, itrequires only 300 visitors on each day for its three shows. This works out to 10,800visitors for 360 days. If each of these persons attends five movies a year, thetheater has to rely on just 2160 regular clients, which should not be too difficultfor a downtown theater of 50 year vintage. So, there seems to be no justification forthe manager’s apprehension that the theater will have to close its doors for ever.

Finally, assuming that high admission rates for new movies does pose a challengeto its survival, one possible solution is to screen time-honored movie classicals andoccupy a special niche in the movie theater market without any additional capitalcost. The manager has not examined this possibility before uttering his doomsdayprediction that ‘it will have to close its doors for ever’.

In anticipating a threat to Rialto’s share of moviegoers from Apex, the managerassumes that, when the same film is being screened in both theaters, residents of thedowntown area would be willing to incur additional expenditure to travel to the malloutside the town, just in order to enjoy its improved features. This assumption goesagainst the result of the survey he has himself quoted which emphasized the cost-consciousness of most respondents.

American Education Aids - GRE Arguments - Model Essays 36

For these reasons, I am not inclined to agree with the doomsday prediction of themanager.

44. Monarch Books and Regal BooksIn this note the business manager of Monarch Books asserts that opening a cafe

would ‘clearly attract more customers to the book shop’, but he does not substantiatethis assertion with any evidence. He does not even say that the rival, Regal Books,has started attracting more customers after it opened its cafe.

He himself enumerates the strengths of Monarch Books as its ‘having been at thesame place for over twenty years’, ‘having a wide selection of books on all subjects’and ‘having a large customer base’. His presumption that these strengths will benullified just because its rival had opened a cafe in its premises sounds thoroughlyillogical. After all, people go to book shops to buy books and not to take coffee!

In fact, there is an inherent contradiction in his note itself. If the cafe isgoing to be attractive to its present ‘large customer base’ and will also attractadditional customers, then the cafe will need quite a substantial area within theshop, and not a ‘relatively little space’. Does he want the main business of the shopto change from selling books to selling coffee?

In support of his recommendation to discontinue the children’s book section, thebusiness manager cites the last national census which indicated a ‘significantdecline in the percent of the population who are under age ten’.

Two questions arise here: If the census had also shown an increasing trend inpopulation growth, then, even with a ‘lesser percentage’ of the total population, the‘absolute number’ of children is likely to increase, and the demand for children’sbooks will not decline.

Secondly, is the national trend applicable to the population in the town in whichMonarch Books is located? It is quite likely that, because of economic, ethnic orother reasons, the overall national pattern is not reflected in the local population,and the percentage of children among the population is on the increase in thisparticular city.

Even if we agree with the business manager’s view in this regard, thediscontinuation of the children’s section will result in some reduction in itscustomer base, and they may be lost to the rival book shop which may continue tooffer children’s books. Customers who switch over to the rival for purchase ofchildren’s books may also decide to buy books on other subjects there. Will this lossbe compensated by the additional customers who, according to the business manager,are likely to be attracted by the availability of coffee here?

Thus, the business manager’s recommendation abounds in a number of lacunae, andshould not be readily acted upon by the management.

45. Arctic deer’s migration patternsWe learn from the first part of the editorial that Arctic deer live on plants

growing on islands which are warm enough to sustain them. During winter, after theseplants are foraged fully, they migrate to a nearby island where the plants are stillalive, walking over the ice between them.

The author then gives the factual information that, according to local hunters,Arctic deer populations are declining. He concludes that, since these reportscoincide with recent warming trends that have caused the sea ice to melt, the declinein the deer population must be because they are no longer able to move from oneisland to another.

The first flaw in his reasoning is that he believes that since phenomenon A iscontemporaneous with phenomenon B, the former must have caused the latter. He doesnot consider the possibility that these two are independent of each other, and thatthere might be other causes for the latter.

For example, the decline in the deer population might well be due to overhuntingor due to some new disease having affected them. The author does not consider thesepossibilities or rule them out with valid reasons.

There is an inherent contradiction in his reasoning. If the global warming has

American Education Aids - GRE Arguments - Model Essays 37

resulted in the melting of the ice between islands, it must have equally enabledthese islands to become warm enough to grow more plants. In such a case, there wouldno longer be any need for the deer to migrate from island to island. If so, themelting of ice would not have made any difference to their population size!

It is also likely that the statement of the local hunters itself is wrong. Theyare probably looking for the deer in those relatively warmer islands to which theyused to migrate before the global warming trends started. If there has been no longerany need for the deer to migrate to these islands because their original habitatsthemselves have become warm enough to sustain the plants on which they forage, thehunters’ conclusion that their population has decreased would be wrong. The author’sexplanation of such decline would then be equally wrong.

For the above reasons, I am not convinced by the conclusion in the editorial.

46. Labrador’s black bearsThe author says that the only report of a grizzly bear ever having been sighted

in Labrador is that of a lone explorer of the nineteenth century who ‘had startledand narrowly escaped from a grizzly bear deep in the woods of that province’.

Since black bears and grizzly bears are similar in colour and the latter is onlymuch larger, this explorer probably mistook, in the darkness of the deep wood, afully-grown black bear to be a young grizzly bear. Note that there is no mention inhis report of the probable age of the bear that he had encountered.

The author then gives the information that ‘modern scientists find no physicalevidence that grizzly bears have ever lived in Labrador’.

He than says that, because the language and legends of Innu, a native tribe ofLabrador, refer to two different types of bears with two different characteristics,grizzly bears had probably lived in Labrador once.

I would take this conclusion with a pinch of salt. The conclusion of scientists(apparently zoologists) on the basis of overwhelming evidence directly relevant tothe question cannot be pooh-poohed on the basis of mere linguistic evidence which isonly marginally germane to the question.

In fact, every language has words for demons and ghosts, and the legends of everysociety refer to them. But does it mean that ghosts and demons do exist and have beenseen by the people?

My own language has words for names of animals and birds which do not live in oursurroundings and have never lived in them. Am I to believe merely on the basis ofthis fact that these animals and birds had once been denizens of our area?

Note that, even while referring to the linguistic evidence, the author does notassert that the characteristics attributed in the legends of the Innu to the twotypes of bears correspond to those of the present-day black bears and grizzly bearsrespectively. There are now three possibilities: (i) the legendary characteristics donot correspond to those of either the black bear or the grizzly bear, in which casethe legends are mere chimera; (ii) one set of characteristics correspond to those ofthe black bear while the other does not correspond to those of the grizzly bear, inwhich case the latter animal was a totally different type of bear which has becomeextinct now; or (iii) the two sets of characteristics correspond to those of theblack bear and the grizzly bear respectively, in which case there could be some basisfor the author’s view. It is by ruling out the first two contingencies withconvincing reasons that the author can persuade me to agree with him.

But, as it stands, the evidence quoted by the author cannot lead me to agree withhis speculation that grizzly bears had once lived in Labrador.

47. Scientists and historical weather patternsThe author first says that the cooling of the earth in the mid-sixth century

could have been due to either a large meteoric collision or a huge volcanic eruption,either of which could have caused a large dust cloud which blocked enough sunlight.He says that few historical records of that time survive now.

He then says that a large meteorite collision would probably create a suddenbright flash of light which none of the existing historical records of the time

American Education Aids - GRE Arguments - Model Essays 38

mention. He adds that some surviving Asian historical records of that time refer to aloud boom which is consistent with volcanic eruption. On this evidence, he speculatesthat the cooling was probably caused by a volcanic eruption and not by meteoriccollision.

The first flaw in the author’s argument is that he considers only twopossibilities - meteoric collision and volcanic eruption - as the cause of thecooling of the earth. Another possibility one can think of is some significant changein solar activity, but the author does not consider it.

Secondly, he himself says that a meteorite collision would ‘probably’ cause asudden bright flash of light. Since he is himself not certain about this result,discarding the theory of meteoric collision just because there is no record of thesighting of a bright flash of light is not logical. Probably this was recorded bywestern observers but those records have been lost.

A meteoric collision is certainly capable of producing a loud boom, and if it hadoccurred without a flash of light (a possibility not ruled out by the author), theavailable Asian records would still support the meteoric collision theory.

Even assuming that a meteoric collision would necessarily cause a bright flash oflight, it is likely that the collision took place in the western hemisphere, and thebright flash of light was not visible in Asia. This could account for the fact thatthe available Asian records do not mention it. Since sound travels in waves, the boomcould have been heard while the light itself was not seen.

So, the author has not provided sound reasons for ruling out the possibility of ameteoric collision as the cause for the cooling of the earth in the mid-sixthcentury.

48. Corpora citizens’ physical fitness Two views are mentioned in this article. One is that of the national advisory

board on physical fitness, and the other is that of the author of the article.The board must clarify whether the standards for adequate physical fitness

defined twenty years ago continue to be valid even now. If the present standards aremore stringent, then a one-to-one comparison between the percentages is not logical.

Secondly, the age composition of the population also plays a role in the averagephysical fitness. If the present-day population has a significantly higher percentageof citizens older than 65 than twenty years ago, then also the average physicalfitness level would have come down, whether or not computers had become popular.

So, the assumption of the board that average physical fitness has declined isitself open to question.

Even assuming that the level of physical fitness has indeed come down in twentyyears, the board has not discussed other possible reasons and ruled them out beforeholding computers to be the main culprit.

The author contradicts the board’s theory by pointing out that, in the regionswhere the levels of computer ownership are the highest, the fitness levels are alsothe highest. He does not consider the possibility that owners of computers usuallybelong to the more affluent sections of the population and, because of betternutrition, their fitness levels are likely to be high. Moreover, most of the presentusers of computers belong to the new generation and are relatively young. So, theaverage age of the population in a region where the levels of computer ownerships arethe highest is likely to be less than in other areas. This could also be anotherreason for their above-average physical fitness. Probably their fitness levels wouldhave been still higher if they were not spending too much time with their computers.So, the author’s contradiction of the board’s view does not have too much forceeither.

From the fact that there has been a reduction in the sale of fitness-relatedequipment and services this year, the author concludes that there has been a recentdecline in the economy of the country and that this has caused the reduction in thelevels of physical fitness.

The first flaw in this argument is that a twenty-year trend is sought to beexplained away by a temporary one-year-old phenomenon.

American Education Aids - GRE Arguments - Model Essays 39

Secondly, considering the sale of fitness-related equipment and services as anaccurate indicator of the vibrancy of a national economy is open to question. Theauthor must have considered and ruled out any other reason which could have caused adecline in the sale of fitness equipment.

As they stand, neither the view of the board nor the view of the author of thisarticle seems to have much logical force.

49. Acne and other skin diseasesTwo reasons are given by the writer of the letter for his assertion that ‘health

experts should no longer recommend that people should avoid eating greasy foods’.The first is that, during the same decade while health experts have been giving

this advice to teenagers to avoid eating greasy foods in order to prevent acne andrelated skin conditions, the number of teenagers who sought medical help fortreatment of these skin diseases has actually risen.

The first flaw in this argument is the unwarranted presumption that teenagers hadfollowed the advice of the health experts and had indeed avoided eating greasy foods.If they had not followed the advice and had continued to consume the same amounts ormore of greasy foods, the increase in the number of reported acne cases wouldstrengthen, and not weaken, the advice of the experts.

The writer has not considered the possibility that, because acne was so commonamong teenagers in the past, it was not considered to be a medical problem, and thatvery few of them consequently used to seek medical treatment for it. Now, because ithas become comparatively rare because of the adherence to the advice of medicalexperts, those who have developed acne and related skin conditions (either because ofnot having followed the advice or due to genetic or other reasons) are seekingmedical treatment. If this would explain the greater number of teenagers seekingmedical treatment for acne than a decade ago, the expert’s advice would in fact bestrengthened and not weakened.

The second reason cited by the writer is that, in a recent study, teenagers whoavoided greasy foods for a month reported approximately as many outbreaks of acne andrelated skin conditions as did those who ate an average of two servings of greasyfood per day. The more logical inference one can draw from this experiment is thatsusceptibility to acne does not vanish as soon as one starts avoiding greasy food,but one should be consistent in such practice and avoid consuming greasy food over aperiod of time. The conclusion of the writer would have been more convincing if theexperiment had lasted for over six months to a year and the result was the same.

Even if one agrees with the author that there is no causal connection between theconsumption of greasy food by teenagers and their developing acne and other skindiseases as a consequence, the logical conclusion can only be that experts need nolonger scare teenagers with warnings against the consumption of such food. Why doeshe generalize this and assert that experts should no longer recommend that ‘people’avoid such foods. If the consumption of greasy food by adults, and particularly bythose beyond the age of 55, is likely to lead to other health problems such as thethickening of the arteries, or blood pressure, or colon cancer etc, the experts wouldbe well within their right (and duty) to issue such warnings.

For the above reasons, I would not give up avoiding greasy food if I were ateenager and did not want my face to be disfigured by acne.

50. Earth and the origin of its oceansIn this article, the author concludes that the water in earth’s oceans must have

originated from comets which had collided with it. I find his conclusion unconvincingfor many reasons.

The author himself concedes the possibility of there having been water vapor evenduring the formation of the earth, arising out of the collision of space rocks. Hesays that some water vapor might have escaped into space in the earliest stages whilethe earth had not developed sufficient gravitational force; but, in a later period,when the earth grew into its present size and developed greater gravitation, thegases and water vapor continued to be retained in its atmosphere.

American Education Aids - GRE Arguments - Model Essays 40

He then advances the argument that the water in earth’s oceans ‘must’ haveoriginated from comets striking it at a later stage of its development because cometsare largely ice made up of frozen water and gases.

There are two problems with this argument.First of all, if the gravitational energy even in the formative stages of the

earth was strong enough for even the surface to get molten, why did the water vapourthat was present not also stick to the earth but flew off into space? A gravitationalforce that was strong enough to retain metals must have been strong enough to retainthe much lighter water vapour also.

First, if the water vapor resulting from the collision of a comet could fall asrain on the earth’s surface, why does he rule out the possibility of water vaportrapped earlier also behaving in the same manner and contributing to the watercontent in the oceans?

Secondly, the author has not quantified the amount of water vapor that a singlecomet collision can give rise to. How many comets must have collided with the earthin order to account for all the water that is contained in the oceans of the earth?Is there any other corroborative evidence for that many number of collisions?

Another shortcoming in the author’s theory is that it does not account for thesalt content in earth’s oceans.

These points must be clarified before the author’s views can be allowed to formpart of a school textbook.